Sunteți pe pagina 1din 51

OPHTHALMOLOGY FINALS TRANS by VIRATA AND SANTOS

Tips in ophtha: Memorize, memorize memorize especially the If tinanong sa exam: What is the main refractive area of the eye?
pictures and know the management in each diseases! Ans. Both Lens and Cornea, pero mas malakas nga lang yung
cornea.
ANATOMY OF THE EYE
THE GLOBE 5 layers of the cornea: ABCDE
• The globe, or eyeball, is NOT a true sphere, it is an Oblate • A-nterior Epithelium
spheroid o Outermost layer
• AP diameter: 23-25mm o Hydrophobic due to its lipid content
o <23mm- short eyeball seen in hyperopias • B-owman’s membrane
o >25mm- long eyeball seen in myopia o Acellular, therefore, cannot regenerate after injury
o Layer that opacifies or forms a scar after injury
THE EYE o Serves as a barrier to most molecules
● 3 Concentric Layers • C-orneal stroma
o Cornea – anterior; Sclera – posterior(white portion) o Thickest layer filled with collagen fibers.
▪ The outermost layer of the eyeball. o Contributes to 90% of the corneal thickness
o Uvea – iris, ciliary body, choroid o Composed of fibroblasts making the cornea
▪ The pigmented layer of the eyeball. optically clear
o Retina – continuous with the optic nerve posteriorly o The regular arrangement of the collagen fibers in
▪ Innermost layer of the eyeball the stroma allow for its optical clarity (98% of light
CONJUNCTIVA is transmitted by the cornea)
• 3 zones • D-escemet’s membrane
o Palpebral conjunctiva- lines the inner aspect of the o Thin layer in which the endothelium attached.
eyelids o Basement membrane of the corneal endothelium
o Bulbar conjunctiva- Covers the sclera o Hassal-Henle Bodies
o Fornix- junction of the palpebral and bulbar ▪ Metabolic by-products of the Descemet’s
conjunctiva membrane
• Vascular supply: Anterior ciliary artery o Corneal Guttata
• Innervation: 1st division of CN V (ophthalmic division) ▪ Central excrescences (look like bubbles
in the central cornea)
• Composed of:
▪ Fuch’s Endothelial Dystrophy-
o Conjunctival epithelium- stratified, columnar
Endothelial cell loss leading to loss of
epithelial cells
visual acuity
o Conjunctival stroma
▪ Adenoid layer - Lymphoid tissue - Does • E-ndothelium
not develop until 2nd or 3rd month of life ▪ Most important layer because it contains the
▪ Accessory lacrimal glands (Glands of endothelial cells
Krause) - Mostly found at the lower fornix ▪ Provides the relative deturgescence or
dehydration of the cornea
TENON’S CAPSULE ▪ Derived from the Neural Crest (neuroectodermal
• Fibrous membrane that envelops the eye from limbus to the in origin)
optic nerve LIMBUS
• Contributes to the “Check Ligaments” - Limits the action
• The junction between cornea and sclera
of the EOM’s
• May develop whitening as part of old age
• Forms the “Lockwood’s Ligament”- Suspensory ligament
o Arcus senilis- often mistaken as cataract
of the globe.
• Importance: serves as a surgical landmark
EPISCLERA
• Covers the sclera INTERNAL OCULAR STRUCTURES
• Provides nourishment for the sclera because the sclera ANTERIOR CHAMBER
is avascular
• Space between the cornea and iris
SCLERA • Contains a watery fluid called aqueous humor
• Covers the POSTERIOR 4/5 of the surface of the globe, • Important structures:
• Avascular, fibrous, outer, protective coating of the eye o Schwalbe’s line- Marks the termination of the
• Continuous with the cornea anteriorly and with the dural corneal endothelium
sheath of the optic nerve posteriorly o Schlemm’s canal and the trabecular
• Lamina cribosa - scleral fibers that pass through the optic meshwork- Drains the aqueous from the anterior
nerve; acts like a sieve chamber
• Where the EOM’s insert o Scleral spur- Inward extension of the sclera
between the ciliary body and Schlemm’s canal
▪ Where iris and ciliary body are attached
Cornea
• Avascular and Transparent UVEA
• Relative deturgescence (78% water content) • Middle, vascular layer of the eye
• Main Refractive Element of the Eye: Power of the eye • Contributes to the blood supply of the retina
(1/3 from the lens) • Composed of three parts:
• Cornea is NOT a perfect circle. It is a Prolate. o Iris
o Ciliary Body

TRANS BY: VIRATA and SANTOS Page 1 of 51


OPHTHA FINALS TRANS

o Choroid - The red-orange reflex is actually coming • Provides blood supply to the outer retinal layers =
from the choroid Nourishes the outer retina
• Bound by:
Iris o Internally - Bruch’s membrane

Most anterior extension of the uveal tract o Externally - Sclera

Function: Screens out light • Suprachoroidal space

Pigmented posterior surface o A potential space between the choroid and the

Blood supply: Long posterior ciliary arteries sclera

Innervation: ciliary nerves

2 muscles: LENS
o Sphincter - Oculomotor nerve controls the contraction
of the sphincter • Biconvex, avascular, transparent
o Dilator – Parasympathetic nerve controls the dilator • Function: Bring images into the retina
muscle. • Composition:
Pupil o 65% water
o 35% protein
• Circular opening at the center of the iris
o Highest protein content of any
• Adjusts the amount of light entering the eye tissue in the body
• Pupillary size is determined by the balance between: o Trace minerals (Na, Cl, K,
o CONSTRICTION due to parasympathetic activity ascorbic acid, glutathione)
(CN III) • Structure
o DILATION due to sympathetic activity o Lens Capsule- Basement
Membrane
o Nucleus: Dense, older lens fiber
o Cortex- less dense, younger lens fiber
• Zonule of Zinn- Composed of fibrils that arise from the
ciliary body
o Functions:
▪ Inserts at the lens equator
▪ Holds the lens in place
▪ Lens thickness changes during
accommodation

MUST KNOW
Ciliary Body
• Consists of 2 parts
o Pars plana (Posterior)
▪ Relatively avascular→ Safest posterior surgical
approach to the vitreous cavity from the corneal
limbus
o Pars plicata (Anterior)
▪ Richly vascularized → avoided in surgery; prone to
bleed.
▪ Consists of ciliary processes
• 2 Principal functions
o Aqueous humor formation • Looking at NEAR: the ciliary muscle CONTRACTS→
o Lens accommodation thru contraction of the ciliary releases tension on the zonules (structure that holds the
body lens) = thickened lens increasing its biconvexity allowing
▪ Changes the tension of the zonular fibers you to focus for near objects = Accommodation
suspending the lens • Looking at FAR: the ciliary muscle RELAXES→zonules
▪ Change in tension leads to increased thickness of are tight, pulling the lens on both sides = flattened lens=
the lens -> allows eye to focus at near objects Relaxation
(accommodation)
• Lined by: ACCOMODATION
o Internal non-pigmented layer- Represents the
• Ability of the lens to change shape
anterior extension of the neuroretina
• 3 components of accommodation:
o External pigmented layer - Represents the
o Thickening of the lens
extension of the retinal pigment epithelium
o Pupillary constriction
• Ciliary muscle
o Midline movement of the eyeball
o Circular muscles which contract and relax the
zonular fibers POSTERIOR CHAMBER
o Alters the tension on the capsule of the lens and ● Space behind the iris and in front of the vitreous
allows the lens to focus for distance and near VITREOUS CAVITY
CHOROID • Transparent gel composed of water, collagen and
• Vascular, pigmented layer between the sclera (externally) hyaluronan
and the retina (internally) • Occupies 80% of the volume of the eye

TRANS BY: VIRATA AND SANTOS Page 2 of 51


OPHTHA FINALS TRANS

VITREOUS HUMOR 4 parts:


• Comprises 2/3 of the volume of the eye • Intra-ocular (optic nerve head)
• Involved in metabolism which serve as a passageway for o Seen as the optic disc on fundoscopy
metabolites used by the lens, ciliary body and retina o Non-myelinated axons
• Intra-orbital
RETINA o Lined by dura, arachnoid and pia mater
o Fuses at the apex of the orbit with periosteum and
● A thin, transparent structure that develops from the inner and
Annulus of Zinn
outer layers of the optic cup
● Lines the vitreous cavity • Intra-osseous/Intra-canalicular
● Ends at the Ora Serrata o Firmly anchored to the bone - Significance: a small
● Functions: mass lesion in the canal can cause compressive optic
o Vitamin A metabolism neuropathy even before it becomes radiological visible
• Intra-cranial
o Maintenance of the outer blood-retinal barrier
o Region where the carotid artery bifurcates into ACA
o Phagocytosis of the photoreceptor outer segments and MCA
o Absorption of light
Remember the different parts:
Layers of the Retina: (from the innermost going outward) • INTRAOCULAR – 1mm
(1) Internal limiting membrane- Abuts the vitreous cavity • INTRAORBITAL – 25mm
(2) Nerve fiber layer- where axons of the ganglion cells found • INTRACANALICULAR – 9mm
(3) Ganglion cell layer- where nuclei of ganglion cells found
(4) Inner plexiform layer- Axons of the bipolar and amacrine • INTRACRANIAL – 16mm
cells and dendrites of the ganglion cells and their synapses • Longest to shortest: orbital > cranial > canalicular > ocular
(5) Inner nuclear layer- Nuclei of bipolar, horizontal, Mueller
and amacrine cells
(6) Outer plexiform layer- Made up of the connections b/w
photoreceptor synaptic bodies and the horizontal and
bipolar cells and is thicker at the macula (layer of Henle)
(7) Outer nuclear layer of photoreceptor nuclei- Rods and
cones
(8) External limiting membrane
(9) Layer of rods and cones
(10) Retinal pigment epithelium (RPE)
o Approximately 1.2 million nerve fibers are found in
the retina.
o Adjacent RPE cells - attached to each other by
junctional complexes which provide both structural
and metabolic stability (outer blood retinal barrier)
o THICKEST at the Papillomacular Bundle
o THINNEST at the Fovea
Macula OPTIC DISC
• It is located temporal to the optic • Portion of the optic nerve visible from within the eye
disc. • Composed of axons whose cell bodies are located in the
• Responsible for central vision ganglion cell layer of the retina
• Fovea
o Thinner outer nuclear ORBIT AND ADNEXAE
layer • Bony, pyramidal cavity that houses the globe, extraocular
o Xanthophyll pigment muscles, nerves, fat, and blood vessel
• Foveola: Clinical Significance
o Central depression within 1. Orbital Roof
the fovea • The fossa for the lacrimal gland – Orbital plate of frontal
o Responsible for the most acute vision bone (lateral)
• Foveal Avascular Zone: • The trochlear fossa (Fovea trochlearis) – frontal bone
(medial); from the orbital margin
o Capillary-free area
o Site of the pulley of the superior oblique muscle,
o Foveal reflex - light reflection of fovea seen during where the trochlea, a curved plate of hyaline
ophthalmoscopy cartilage is attached
• Lesser wing of sphenoid of the sphenoid- contains the
OPTIC NERVE optic canal
• Not really a peripheral nerve, it is actually a direct extension • If you injure the roof, two things to consider is if the lacrimal
of the CNS gland is injured as well if there is affectation in superior oblique
• Corresponds to the “blind spot” in perimetry muscle.
• Blood supply: Circle of Zinn-Haller 2. Orbital Floor
o Short Posterior Ciliary Artery • Maxillary bone: Blowout fractures – weakest point
o Choroidal circulation • Most common site of fractures because it is considered
o Pial vasculature to be WEAKEST point.

TRANS BY: VIRATA AND SANTOS Page 3 of 51


OPHTHA FINALS TRANS

• Contains the infraorbital groove -> infraorbital foramen Muscle Primary Secondary Tertiary
• Inferior oblique muscle- arises from the orbital floor and the Medial Adduction __ __
only extraocular muscle that does not originate from the Rectus
orbital apex Lateral Abduction __ __
3. Lateral Orbital Wall Rectus
• Least affected in trauma Inferior Depression Excycloduction Adduction
• Thickest and Strongest → because of the zygomatic bone. Rectus /Extorsion
4. Medial Orbital Wall Superior Elevation Incycloduction Adduction
• Thinnest point: lamina papyracea Rectus /Intorsion
• But not weakest: due to ethmoid air cells Inferior Excycloduction Elevation Abduction
• Site of lacrimal fossa (fossa of the lacrimal sac) and Oblique /Extorsion
nasolacrimal canal Superior Incycloduction Depression Abduction
Oblique /Intorsion
Fissures
• Superior orbital fissure Mnemonics:
o Separates the greater and lesser wings of the • Lahat ng Superior, Laging INtorsion
sphenoid • Inferior ang EXtorsion mo
o Cranial nerves III, IV, V1, VI, superior orbital vein • Lahat ng may oBlique laging aBduction
• Inferior orbital fissure
o Cranial nerve V2, inferior orbital vein
NERVE SUPPLY
• CN 6- Lateral Rectus
• CN 4- Superior Oblique
• CN 3- All other muscles and the levator palpebrae
superioris

BLOOD SUPPLY
Ophthalmic Artery
• 1st Major branch of Internal carotid artery after it emerges
from the cavernous sinus
• Branches of the ophthalmic artery
o Central retinal artery – pierces the optic nerve and
enters the eyeball at the center of the optic nerve (or
the optic disc)
o Muscular branches of the extraocular muscles –
gives off the anterior ciliary artery which supplies the
anterior sclera, episclera, limbus, conjunctiva and
contributes to the major arterial circle of the iris
o Posterior Ciliary arteries – anterior and posterior
▪ Long Posterior Ciliary – supply the ciliary body
and anastomose with each other to form the major
arterial circle of the iris
▪ Short Posterior Ciliary – supply the choroid and
EXTRAOCULAR MUSCLE the optic nerve head
• 4 recti- Medial rectus, Lateral rectus, Superior rectus, o Lacrimal artery – supplies the lacrimal gland and the
Inferior rectus upper eyelids
• 2 oblique- Superior oblique and Inferior oblique o Supratrochlear and supraorbital arteries – supply
the skin of the forehead
• 1 levator- Levator palpebrae superioris
Ophthalmic Veins
• Recti muscles – Originate from the Common tendinous
ring/ ANNULUS OF ZINN (orbital apex). Recti muscles • Superior and Inferior ophthalmic vein
insert straight to the eyeball. o Both drains into the cavernous sinus
• Superior Oblique – Originates from the common o Drains the vortex veins, the anterior ciliary veins, and
tendinous ring/ ANNULUS OF ZINN (orbital apex) and the central retinal veins.
inserts in the posterior temporo-superior part of the
eye. It does not go directly to the eye, but it goes up to OCULAR ADNEXA
the trochlea then comes back down to the eyeball. • Soft Tissues
• Inferior oblique – Originates from the MAXILLA (orbital
floor) and inserts at the posterior temporo-inferior part EXTERNAL OCULAR STRUCTURES
of the eye. It starts on the floor then it runs directly to the EYELID
eyeball. Skin and subcutaneous connective tissue
• Levator palpebrae superioris – Originates from the • Thinnest skin in the body
LESSER WING OF SPHENOID BONE and is the only
• Unique → no subcutaneous fat
muscle involved in raising the superior eyelid.
• Eyelid fold- due to insertion of levator aponeurosis near the
Innervated by oculomotor nerve (CN III).
upper border of the tarsus (may not be present in Asians)

TRANS BY: VIRATA AND SANTOS Page 4 of 51


OPHTHA FINALS TRANS

Lid Margin Lacrimal Excretory System


• Punctum “Gray Line” – corresponds histologically to the • Located in a shallow depression within the orbital part of the
most superficial part of the orbicularis oculi muscle -> frontal bone – lacrimal fossa
Muscle of Riolan • Pathway:
o Eyelashes arise ANTERIOR to this line Lacrimal punctum → Upper and lower canaliculi → 1mm
o Meibomian glands (within the tarsus) arise Common canaliculus → Nasolacrimal sac → Nasolacrimal
POSTERIOR to this line duct → Inferior turbinate in the nose.

PROTRACTORS: Orbicularis Oculi Muscle


• Is innervated by facial nerve (CN VII)

Orbital Septum
• Extension of the periosteum of the roof and floor of the orbit
• Attaches to the anterior surface of the levator muscle
• Provides a barrier to spread of blood or inflammation
o Prevents spread of infection to the orbit. So if you
have infection of the eyelid or preseptal cellulitis,
it’s only on the surface.
Orbital Fat
BASIC EYE EXAM
• Lies posterior to the orbital septum and anterior to the • First step is to check the vision first, establish vision before
levator aponeurosis (upper lid) or the capsulopalpebral any intervention for security or legal reasons
fascia (lower lid)
• Basic Terms:
o Oculus = Eye
• Clinical significance: if there is trauma, one thing to check o Dextro = RIGHT
for is the presence of protruding fat. Pagwala, we repair o Sinister = LEFT
it as iclose lang yung wound. o Uterque = BOTH
• Pero pag meron prolapsing fat meron tayo idea na violated ▪ OD (oculus dexter) = right eye
ang septum. If we don’t repair this one, ang problem ay ▪ OS (oculus sinister) = left eye
yung fats will not return to its normal position, ▪ OU (oculus uterque) = both eyes
magkakaaroon tayo ng eyebags.
VISUAL ACUITY
Retractors
• Measurement of the smallest object a person can identify at
• Upper eyelid– Levator muscle with its aponeurosis and the a given distance
superior tarsal muscle (Muller muscle) at the apex of the o Metric systems (meters) – Philippines
orbit o English system (feet)
o Levator aponeurosis- produces the eyelid fold
o Superior Tarsal muscle (Muller muscle)- elevates the DISTANCE VISUAL ACUITY
eyelid and innervated by superior division of CNIII • Measured as a fraction ex. 20/20
• Lower eyelid– Capsulopalpebral fascia and the inferior • First number (numerator) – distance of patient from eye
tarsal muscle chart
o Different eye charts have different distances. What
Tarsus we usually use is EDTRS chart at 6m or the
• Firm, dense plates of connective tissue that serve as the Snellens at 20ft.
structural support of the eyelids • Second number (denominator) – distance at which the
• Skin of the eye letter can be read by a person with normal vision
• Meibomian glands- modified holocrine glands (oily layer of • 6/6 (metric) = 20/20 English = 1.0
the tear film) SNELLEN CHART CORREPONDING VA PER LINE

Conjunctiva Line Visual Acuity


• Forms the posterior layer of the eyelids and contains the 1 20/200
mucin-secreting goblet cells and the accessory lacrimal
2 20/100
glands of Wolfring and Krause
3 20/70
Other Accessory Eyelid Structures
4 20/50
• Plica semilunaris
o Rich in goblet cells 5 20/40
o Analogous to the 6 20/30
nictating membrane
in the lower animals 7 20/25
• Caruncle 8 20/20
o Contains the
sebaceous glands 9 20/15
and fine colorless
10 20/13
hair
11 20/10

TRANS BY: VIRATA AND SANTOS Page 5 of 51


OPHTHA FINALS TRANS

TESTING DISTANCE VA INTRAOCULAR PRESSURE (IOP) MEASUREMENT


• Position patient at 6 meters • Normal IOP range – 10-21 mmHg
• Start with right eye first (occlude other eye) • Gold standard: Goldmann Applanation Tonometer
• Ask patient to read smallest line he can distinguish • IOP is determine by 3 things:
• If VA is 6/12 (line 5) or worse, repeat test using a pinhole o Production of Aqueous
• Test the left eye similarly o Outflow of removal of Aqueous
o What if the patient can’t read 1 letter. That’s o Episcleral venous pressure
20/40 -1. If 2 letters, that’s 20/40 -2
• Record VA for each eye ANCILLARY TESTS
• If patient cannot see the largest letter: CONFRONTATION FIELD TESTS – VISUAL FIELD TEST
o Bring patient CLOSER • Visual field – the entire area seen when your gaze is fixed
o If still unable to see largest letter at 1 meter, do in one direction
COUNTING FINGERS
o If unable to count fingers, determine if he can DETECT
MOVEMENT OF YOUR HAND AMSLER GRID TEST
o If patient cannot detect movement, determine if he can • Evaluates 10 degrees of visual field
DETECT PRESENCE AND DIRECTION OF LIGHT • A test of central vision
▪ LP – light perception • Detects macular diseases
▪ LP with direction – light perception with • For macular edema, patient sees wavy lines
localization of light
▪ NLP – no light perception (totally blind)
PINHOLE TEST
• To assess best corrected vision
• By looking through a pinhole, the
refractive errors of the peripheral
cornea and crystalline lens of the
eye are significantly reduced or
eliminated
• Those who can see the chart but
blur, use pinhole. If vision improved
by looking to pinhole patient has
REFRACTIVE ERROR (may grado
ang mata).
Macular Edema Normal
NEAR VISION TESTING
• Done in patients with: FLOURESCEIN DYE STAINING
o Near vision complaints
o Difficulty testing for distance (ex. Bedridden
patient)
o Age of 40
• Rosenbaum pocket vision chart
o Done at the recommended distance (usually 13-15
inches in front of the patient)
o Test one eye at a time
o Measurement is recorded as a Jaeger equivalent
o A person with a normal vision can read Jaeger 1
o A person who cannot read Jaeger 1 and can only
read up to Jaeger 5, usually needs corrective
glasses Dendritic Ulcer (HSV)
COLOR VISION TEST
PUPILLARY REACTION TESTING
• Checks patient’s ability to distinguish colors
• Direct pupillary reaction – constriction of pupil when
• ISHIHARA CHART
tested eye is exposed to bright light
• Consensual pupillary reaction – constriction of the pupil
of the opposite eye when tested eye is exposed to bright
light

SWINGING PENLIGHT TEST


● Indication: Anisocoria (Unequal pupil)

Normal response:
• Direct light reflex – pupil constricts to direct light
• Consensual light reflex – pupil constricts to light shining on
opposite pupil

TRANS BY: VIRATA AND SANTOS Page 6 of 51


OPHTHA FINALS TRANS

OPHTHALMOSCOPY Emmetropia
LANDMARKS • Absence of refractive error.
• (+) Red-Orange Reflex • Image focused clearly on the fovea/retina
• Optic Disc- Physiologic Cup (0.2-0.4)
o >0.5 seen in glaucoma Ametropia
• Normal Artery-Vein Ratio- 2:3 • Any refractive error of the eye which prevents normal
focusing of the image.
OPTICS AND REFRACTION
VISUAL SYSTEM Anisometropia
• Difference in refractive error between the two eyes. It is a
major cause of amblyopia (lazy eye) because the eyes
cannot accommodate independently and the more
hyperopic eye is chronically blurred.

Aniseikonia
• Difference in the size of the retinal images.
The eye as the camera. • Predominantly due to monocular aphakia (which can be
congenital or iatrogenic after a surgical complication).
Eye Camera Symptoms of Refractive Errors
Cornea Lens Filter • None – especially in children
Lens Lens • Slowly progressive blurring of vision
Retina Film • Squinting
Iris Diaphragm • Headaches
Pupil Aperture • May have previous history of eyeglass use
Sclera Camera Body

BLURRED VISION Signs of Refractive Errors


• Most common eye complaint • Visual acuity that improves with pinhole
o We are referring to painless, or otherwise • Pinhole blocks nonparallel rays of light that
asymptomatic BOV. cannot be focused on the retina
• Eye exam is generally normal
Serious eye disease if associated with the following • Myopic patients may have:
SYMPTOMS: • Tilted disc
• Eye pain • Myopic crescent – they can be a sign of
• Light sensitivity other diseases Myopic crescent
• Flashes of light
• Floaters TYPES OF REFRACTIVE ERRORS
• Distortion of vision (metamorphopsia) MYOPIA (NEAR-SIGHTEDNESS)
• Severe visual field loss • The myopic person has the advantage of being able to read
 Consult an ophthalmologist if above symptoms are at the far point without glasses even at the age of
present. presbyopia.
• Causes
Serious eye disease if BOV is associated with the following o Cornea is too steep (curvature myopia or
SIGNS: refractive myopia.
• Eye redness o Eyeball is too long (axial myopia)
• Visual field defect • Effect: light rays fall in front of the retina
• RAPD (Relative Afferent Pupillary Defect)
• Abnormalities of the cornea, iris or pupil HYPEROPIA (FAR-SIGHTEDNESS)
• Absence of red reflex • Patient is able to see objects in the distance but not 20/20
• Optic disc swelling/pallor • Patients usually require bifocal to correct both far and near
vision
REFRACTIVE ERRORS • The hyperopia that cannot be corrected by accommodation
• Mismatch between the curvature of the cornea and the is termed manifest hyperopia. This is one of the causes of
length of the eyeball deprivation amblyopia in children and can be bilateral.
• Inability of the eye to focus an image on the fovea or central • The degree of hyperopia overcome by accommodation is
retina known as latent hyperopia. It is detected by refraction after
• Refractive errors are inherited. instillation of cycloplegic drops, which determines the sum
• In general, myopic eyes, have highly powerful optics such of both manifest and latent hyperopia.
that the image focuses in front of the retina because the • Causes:
focusing power of either the lens or the cornea is so o The cornea is too flat (refractive hyperopia)
malakas. Whereas in hyperopes, the image is focused o The eye is too short (axial hyperopia)
behind the retina. • Effect: light rays fall in focus behind the retina

TRANS BY: VIRATA AND SANTOS Page 7 of 51


OPHTHA FINALS TRANS

ASTIGMATISM PHYSICAL EXAMINATION


• The eye produces an image with multiple focal points APPLANATION TONOMETRY
or lines. • Gold standard for determining the intraocular pressure
• In regular astigmatism, there are two principal meridians, • Measures the IOP based on the following equation:
with constant power and orientation across the pupillary o P=F/A
aperture, resulting in two focal lines. o Intraocular pressure (P) is equal to the force (F)
• In irregular astigmatism, the power or orientation of the necessary to flatten the cornea divided by the area
principal meridians changes across the pupillary aperture. (A) flattened.
• Causes:
o Cornea is steeper in one meridian/axis than in
another GONIOSCOPY
o The eye is not a perfect sphere (oval or almond- • For viewing the angles
shaped) • Is an eye examination to look at the front part of your eye
• Effect: image is sharper in one plane/axis causing blurred (anterior chamber) between the cornea and the iris.
vision and distortion of viewed images • To see whether the area where fluid drains out of your eye
(called the drainage angle) is open or closed.
• For you to be able to say that an angle is open, you need
PRESBYOPIA
to see at least the posterior trabecular meshwork. (the
• Difficulty reading up close (less than one arm’s length) brown layer)
• The loss of accommodation that comes with aging to all
people is called presbyopia =
• Cause:
o With age, there is a natural tendency of the human
crystalline lens to harden limiting the ability of the
eye to focus on near objects (accommodation)
o By age 40, viewing of near objects becomes
difficult
Symptoms and Signs:
• Normal distance vision (corrected vision)
• Poor near vision
• Normal eye exam (Left) Open Angle, (Right) Close Angle

Management of Refractive Errors: VISUAL FIELD DEFECTS IN GLAUCOMA


• Myopia- Concave (minus) lenses • You have to remember that any lesion that you see in the
• Hyperopia- Convex (plus) lenses retina, the impairment in the patient’s visual field will always
• Astigmatism- Cylindrical lens be inverse or reverse.
MUST KNOW! o E.g. If you see a problem in the nasal retina, the
patient sees the defect in the temporal retina. If
RECOMMENDED VISION TESTING you see the defect in the inferior, the patient sees
• Vision screening should be done at each well child visit the defect in the superior.
o Age 3-4 - VA testing should be performed • In glaucoma, the lesion is usually localized in the horizontal
o Age 5 - VA & motility testing should be performed midline and the blind spot corresponds to your optic
o At age 40 - VA tested every 2-4 years until age 65 nerve
o At age 65 - VA tested every 1-2 years o In a Visual Field Exam, kung anong side makikita
yung blind spot, yun yung laterality ng eye mo.
GLAUCOMA Kung nasa left side yung blind spot, ibig sabihin
• Refers to a group of diseases that have certain common yun yung left eye mo.
features: • Central scotoma is not seen in glaucoma.
o Cupping of the optic nerve (ON) head- signifies
death of the retinal ganglion cells 1. Paracentral Scotoma
o Visual field loss- Due to structural changes such
as death of the retinal ganglion cells and thinning
of the Retinal Nerve Fiber Layer
• In Glaucoma, there should be STRUCTURE (optic nerve
cupping) and FUNCTION (visual field defect) correlation.
• Elevated IOP is a primary risk factor.
• Death of retinal ganglion cells can be seen in the optic rim.
(lesion of glaucoma is a retinal ganglion disease) and
Retinal nerve fiber layer (RNFL) thinning
• Initially affected: Peripheral vision
• Most patients are Asymptomatic

TRANS BY: VIRATA AND SANTOS Page 8 of 51


OPHTHA FINALS TRANS

TYPES OF GLAUCOMA
2. Arcuate (Bjerrum) Scotoma ACUTE VISUAL LOSS: ACUTE ANGLE CLOSURE
GLAUCOMA
Classification by Clinical Presentation AACG:
• Acute
• Subacute/ Intermittent – closes for a limited period, then
spontaneously resolves
o Usually presents with a normal IOP, narrow angle
and signs of a previous attack
• Chronic – an acute case that has not been treated or
resolved
o Presents with an IOP 50 and below, less pain but
otherwise clinically similar to an acute case

Symptoms of AACG:
• severe headache
• eye pain -caused by the stretching of the ocular tissues
• eye redness -due to congestion of the blood vessels
3. Nasal Step Scotoma- earliest sign of optic nerve
• blurred vision - caused by corneal edema and nerve
damage
damage in glaucoma
• colored haloes around lights -caused by light refraction
from corneal edema
• nausea and vomiting -Response to pain

Signs of AACG:
• Congested vessels in the conjunctiva and episcleral-
internal eye pressure is greater than blood pressure
• Mid-dilated, non-reactive pupil- due to sphincter paralysis
+/- posterior iris adhesion to the lens
• Shallow anterior chamber- due to a build-up of pressure
behind the iris
• Corneal clouding- due to corneal edema
• Hard eye- IOP rises to above 40mmHg

Epidemiology of AACG:
• Eskimos (HIGHEST)
• Blacks (LOWEST)
4. Temporal Wedge Defect • Peak incidence 55 - 65 yr old
• Women
• Polygenic inheritance

Anatomical Characteristics of Eyes Prone to AACG


• Short Axial Length
• Anterior Segment Crowding
• HYPEROPES (small eyeballs, AP dimension of the eyeball)

Differential Diagnosis of Closed Angle Glaucoma


• Nanophthalmos
o Think small – “uNANOngmata”
o Bilateral condition
o Globes are shorter
o Axial length of less than 20 mm
o Severe hyperopia – (a result of short globes)

Management of AACG:
MEDICAL:
The most common of these are tunnel vision and arcuate • Oral carbonic anhydrase inhibitor
scotoma. o Acetazolamide 250 mg, 2 tablets then 1tab q6
• Intravenous 20% mannitol 1-2 g/kg BW
• In acute angle closure glaucoma, the only medication that
will work are acetazolamide and mannitol.

TRANS BY: VIRATA AND SANTOS Page 9 of 51


OPHTHA FINALS TRANS

SURGICAL: 3. Normal Tension


• Laser treatment o “Low tension glaucoma”
o Laser Iridectomy – considered the DEFINITIVE o Chronic open angle glaucoma with optic nerve damage
TREATMENT and visual field loss associated with normal IOP
• Incisional surgery – for ADVANCED cases o Painless, progressive loss of visual field (even with
o Create a new pathway (fistula) that allows treatment)/optic disc cupping
aqueous humor to flow out of the anterior chamber o Lowering IOP to low teens results in significant
through the surgical opening in the sclera and into progression of visual loss
the subconjunctival and sub-tenon spaces o Worst glaucoma of all. kasi kahit anong baba mo ng
• Trabeculectomy IOP nag dedeteriorate parin ang vision.
o Perform trabeculectomy punch. In
trabeculectomy punch, you are actually picking up Risk factors for Open Angle Glaucoma
a block of your cornea to create an access. • Increasing Age
Through that newly created passage, the aqueous • Race (more common in African-Americans)
humor can now move out of the anterior chamber.
• Family History
The aqueous humor will now go the conjunctival
• Vascular disease: Systemic Hypertension, DM, Thyroid
space where it will be drained by the vessels. You
disease, Migraine
are actually creating a new pathway for the
• HIGH MYOPIA
aqueous to be drained.

Contraindicated Medications: Symptoms of OAG (more of primary type)


• Topical pupil dilators (mydriatics) • Insidious
o pushes the iris back into the angles • Gradual, progressive blurring of vision (peripheral first)
• Sympathomimetic and anticholinergic medications • Eye pain
o Dilates the pupils • Redness
o Antihistamine, cold remedies and antispasmodics • Haloes
• In glaucoma, the peripheral vision is usually affected first, in
What are the chances of the same thing happening to the advanced stages that is when the central vision is usually
other eye? affected.
• 40 to 80% in 5 to 10 years
• *Translation = Do laser iridectomy on the other eye Treatments of OAG
(prophylactic iridotomy) • Suppression of aqueous production
o Beta-adrenergic blocking agents (timolol,
CHRONIC VISUAL LOSS: OPEN ANGLE GLAUCOMA betaxolol, levobunolol
o Alpha-adrenergic agonists (brimonidine,
• Outflow of aqueous humor through the trabecular
apraclonidine)
meshwork- schlemm’s canal venous system is
o Carbonic anhydrase inhibitor (acetazolamide,
impaired
dozolamide)
• Cause of trabecular obstruction is not known
• Facilitation of aqueous outflow
• Open angle glaucoma is insidious, asymptomatic, and o Parasympathomimetic agents (pilocarpine)
more dangerous. Most of the patient will present advance o Prostaglandin analogs (latanoprost, travoprost)
eye disease. o Adrenergic agents (epinephrine)
• Reduction in vitreous volume
Types of Open Angle Glaucoma
• Miotics, mydriatics or cycloplegic
1. Primary Type • Medical: Topical Medications
o “High tension glaucoma” o Beta blockers: -ol
o Most common form o Alpha-2 Adrenergic agonist: Brimodine,
o Insidious, progressive, bilateral visual loss Apraclonidine
o Often not detected until significant visual loss o Carbonic Anhydrase Inhibitors: Acetazolamide
o They weren’t able to note/detect that they have o Prostaglandin Antagonist: Latanoprast.
glaucoma because they are usually asymptomatic Travoprost
2. Secondary Type o Adrenergic Agonist (Epinephrine)
o “High tension glaucoma” o Cholinergic Agonist (Pilocarpine)
o Increased resistance to trabecular meshwork
• Systemic medications
outflow associated with other conditions (steroid-
o Mannitol 20% IV- usually used in severely
induced glaucoma, phacolytic glaucoma)
elevated IOP and in acute angle closure attacks.
o Increased post-trabecular resistance to outflow
Its mechanism is it reduces the vitreous volume. It
secondary to elevated episcleral venous pressure
does not affect the outflow. It mostly
(carotid cavernous sinus fistula, Grave’s disease)
hyperosmotic diuresis, it reduces the water
o Usually secondary to steroid use because steroid can
content of the vitreous to lower down the pressure.
alter the structure of the trabecular meshwork.
o It can also be secondary to inflammation, if the • Surgery: Laser trabeculoplasty. It causes follicular
trabecular meshwork is inflamed it will not work changes in the trabecular meshwork to facilitate outflow.
properly. Remember per MOA (tinanong sa exam!)
o Secondary to trauma, there is destruction of the
structures of the trabecular meshwork so it won’t
function properly.

TRANS BY: VIRATA AND SANTOS Page 10 of 51


OPHTHA FINALS TRANS

CHRONIC VISUAL LOSS


CATARACTS
• Any opacity of the lens
• Common complaint : BOV (decreased VA)
• Common cause of significant visual loss impairment in the
world
• Incidence/Prevalence
o Most comon of age 75-85 have some cataract changes Cortical Cataracts
o Sex: Male= Female
• Posterior Subcapsular cataract
o Formation of granular and plaque-like opacities
TYPES OF CATARACTS in the posterior subcapsular cortex
• Age Related (“senile”)- over 90% (most common)
• Childhood cataracts- Toxic/Nutritional

AGE-RELATED CATARACTS
• Signs and symptoms: IMPORTANT!
o Blurred vision – gradual, progressive
o Distortion or "ghosting" of images Posterior Subcapsular Cataract
o Problems with visual acuity in bright light or night
driving (glare) PATHOGENESIS OF AGE RELATED CATARACTS:
o Second sight: increased myopia • New cortical layers are added in a concentric pattern, the
o Yellowish or brownish discoloration of objects central nucleus is compressed and hardened in a process
o Falls or accidents -Injuries (e.g., hip fracture) called NUCLEAR SCLEROSIS
• Risk factor: AGE • Lens epithelium -
• Eye examination: A lens opacity o Decrease in lens epithelial cell density
• Age-related cataract - if symptoms and ophthalmic exam o Differentiation of lens fiber cells
support cataract as major cause of vision impairment o Decrease in the rate of transport of water,
nutrients, and antioxidants to the lens nucleus
SAMPLE CASE: o Progressive oxidative damage to the lens with
• 68 y/o female aging
• CC: blurred vision • Conversion of soluble low-molecular weight cytoplasmic
• HPI: gradual progressive BOV starting 2 yrs ago lens proteins to:
o (+) glare with bright lights o Soluble high molecular weight aggregates
o Colors not as bright o Insoluble phases
o Near vision improved o Insoluble membrane-protein matrices
o No other ocular complaints ➢ Cause abrupt fluctuations in the refractive index of
• Exam: VA - 6/15 OU the lens, scatter light rays, and reduce
o No obvious whitening of lens transparency
o Fundoscopy: hazy media, fundus details difficult to • Role of nutrition - glucose and trace minerals and vitamins
appreciate
Impression: AGE- RELATED CATARACT CONGENITAL CATARACTS
• Signs and symptoms:
3 TYPES OF AGE-RELATED CATARACTS o Often asymptomatic
• Nuclear cataract o Lens opacity present at birth or within three
o Excessive nuclear sclerosis and yellowing months after birth
o Results in a central lenticular opacity o Visual inattention or strabismus (lazy eye)
o Can become very opaque and brown o Leukocoria (white pupil reflex)
(“brunescent”) o Nystagmus
• Causes:
o Drugs (corticosteroids in first trimester,
sulfonamides, etc.)
o Metabolic- diabetes in mother, galactosemia in
fetus
o Intrauterine infection -first trimester (rubella,
herpes, mumps)
o Maternal malnutrition
Nuclear cataracts • Must always rule out ocular tumor- early diagnosis and
treatment of retinoblastoma may be lifesaving
• Cortical cataract • Important to ask family history and maternal history
o Changes in ionic composition of the lens cortex
o Results in a change in hydration of the lens SAMPLE CASE:
fibers • Newborn female (2nd child)
• Mother noted a white mass on both eyes 3 months after
birth
• Child has normal developmental milestones
• Maternal History:

TRANS BY: VIRATA AND SANTOS Page 11 of 51


OPHTHA FINALS TRANS

o Mother had history of rubella infection during o Usually with implantation of a plastic intraocular
pregnancy lens (IOL)
o Normal spontaneous vaginal delivery o Lens implanted inside the capsular bag
o Non-diabetic • Phacoemulsification
• Family History: o Emulsify the nucleus using ultrasound
o No history of cataracts/ocular tumors in parents o Intraocular Lens is usually inserted
o Other sibling is 4 yrs old with no eye problems o Advantages:
Impression: CONGENITAL CATARACT ▪ Smaller incision size
▪ Less corneal distortion
▪ Faster surgical time
TRAUMATIC CATARACTS ▪ Faster healing time
• Causes:
o Penetrating injury to the eye PREVENTION/AVOIDANCE
o Intraocular foreign body • Use of ultraviolet glasses in sunny climates
o Blunt trauma to eyeball • Antioxidants (vitamins C and E)
o Heat (infrared)
o Electrical shock
o Radiation EXPECTED COURSE AND PROGNOSIS
• Lens becomes white soon after the injury CATARACT PROGNOSIS
Age- Related Cataract Ocular prognosis is good
SAMPLE CASE: after cataract removal if no
• 33 year old male -factory worker prior ocular disease
• CC: rapid BOV on the L eye Congenital Cataract Prognosis is often poor
• HPI: because of the high risk of
o 5 days, patient grinding a metal pipe when he amblyopia
noted FB sensation on the L eye
Other Cataract Types Depends on type but mostly
o FB resolved but pt noted some tearing
with good prognosis if no
o 3 days ago, rapid progressive BOV, L eye
underlying ocular disease
• Exam: – VA OD - 6/6, VA OS -HM
o Normal findings OD
o OS: clear cornea, round pupil, white lens, no view ECTOPIA LENTIS
of fundus (UNILATERAL) • “Dislocated lens”
Impression: TRAUMATIC CATARACTS • Partial (subluxated) or complete dislocation
• (+) Iridodonesis- vibration or agitated motion of the iris with
eye movement
• Causes:
o Hereditary (Marfan’s Syndrome)
▪ Spontaneous subluxation or dislocation
(superiorly or anteriorly)
o Traumatic (boxers, blunt injury)
▪ Posterior dislocation
• Treatment:
Traumatic Cataract o Surgical removal frequently necessary if complete
dislocation
EYE EXAMINATION: o May observe if partial dislocation and VA is good
• Fluorescein retinal angiography – Retinal disease • Complications if lens is not removed
o Glaucoma
o Uveitis
TREATMENT o Retinal detachment
• Outpatient or inpatient surgery
• There is no medication at present to prevent or slow the
progression of cataracts
• Surgical Intervention indicated if:
o Symptoms distressing to the patient
o Interference with lifestyle or occupation
o Risk of fall or injury
o Vision no longer improved with eyeglass
correction
• Except for congenital cataract: Surgery ASAP! To
prevent amblyopia Lens Subluxation vs. Lens Dislocation

SURGICAL TECHNIQUE
• Cataracts are NOT removed by laser!!
• Intracapsular cataract extraction
o Removal of entire lens (rarely done)
• Extracapsular cataract extraction
o Removal of cortex and nucleus only

TRANS BY: VIRATA AND SANTOS Page 12 of 51


OPHTHA FINALS TRANS

ACUTE VISUAL LOSS EXUDATIVE RETINAL DETACHMENT


RETINAL DETACHMENT • Subretinal fluid from the choroid gains access to the
• Separation of the neural epithelium (sensory retina) from subretinal space through damaged RPE
the pigment epithelium of the retina • No retinal breaks, No photopsia
• SHIFTING FLUID
o Choroidal tumors and inflammation

Detached retina : Corrugated appearance and undulates


Subretinal fluid
MAJOR TYPES OF RETINAL DETACHMENT
TREAMENT OF RETINAL DETACHMENT
RHEGMATOGENOUS RETINAL DETACHMENT
• Rhegmatogenous/Tractional →SURGERY
• Caused by a retinal break/ retinal hole
• Exudative RD - Use of oral or systemic corticosteroids
• Most common
• Seen in myopic patients SAMPLE CASE:
• Can also be due to trauma • 40 yo male, myopic
• Symptoms of RRD: • Sudden blurred vision -left
o 60% symptomatic o Floaters and flashes of light 2 days earlier
o Photopsia (flashes of light) - No lateralizing value • VA OD: 20/20 VA OS: 20/100
o Floaters (solitary, cobwebs, sudden shower)
• Fundus: difficulty focusing on the retina, undulations of the
o Visual field defect (Black curtain, “Curtain falling”)
retina
• Impression: Rhegmatogenous Retinal Detachment

VASO-OCCLUSIVE DISEASES
DIABETIC RETINOPATHY
• Pathogenesis:
o Microvascular OCCLUSION
o Microvascular LEAKAGE

MICROVASCULAR OCCLUSION
• Thickening of the basement membrane
Floaters are seen as small specks in your • Capillary endothelial cell damage and proliferation
vision and are usually harmless • Changes in red blood cells
• Increased in platelet stickiness
➢ Result: Defective Oxygen transport
o Choroid and retina = high oxygen demand
➢ Consequence → Ischemia

Before surgery After surgery

TRACTIONAL RETINAL DETACHMENT


• Sensory retina is pulled AWAY from the RPE by
contracting vitreoretinal membranes
• No retinal breaks
o Diabetic retinopathy, Penetrating ocular Arteriovenous Shunts Neovascularization
trauma
MICROVASCULAR LEAKAGE
• Pericytes–responsible for the structural integrity of the
vessel wall
• High blood sugar –Loss of pericytes
o → Distention of capillary walls and break down of
blood retinal barrier (HEMORRHAGES)
o → Leakage of plasma constituents into retina
(EXUDATES AND RETINAL EDEMA)

TRANS BY: VIRATA AND SANTOS Page 13 of 51


OPHTHA FINALS TRANS

DIABETIC RETINOPATHY TYPES:


• Non proliferative
• Proliferative

CENTRAL RETINAL VEIN OCCLUSION


• Thrombosis of the central retinal vein at and posterior to
the lamina cribrosa
Non- Proliferative Proliferative
Remember! Sa proliferative- yung white ay exudates/retinal
edema, yung red ay hemorrhages (Tinanong sa exam)

TREATMENT OF DIABETIC RETINOPATHY


• Treatment Options: Laser and Surgical

CRVO

LOSS OF VISION
Laser Surgical
• Macular edema –most common cause of persistent poor
SAMPLE CASE:
vision
• 48/F with BOV, OD x 3 days
• Macular ischemia
o Large floater blocking her view
• frequent urination, losing weight the past few months
TREATMENT
• Examination: VA OD -20/200, VA OS-20/70
• Ischemic/ non ischemic- Evaluate by fluorescein
• Fundus: numerous hemorrhages and exudates, OU
angiography
o Large hemorrhage on the right eye
o If Ischemic -laser treatment
• Impression: Diabetic Retinopathy
• Manage the systemic co-morbidity (HTN, DM, etc)

RETINAL VEIN OCCLUSION


• 2nd most common retinal vascular disease
• Classification
o BRANCH retinal vein occlusion (BRVO)
o CENTRAL retinal vein occlusion (CRVO)
• Predisposing factors:
o Increasing age 6th –7th decades
o Systemic hypertension (65-75%) *High risk CRVO- marked loss of visual field, Marcus gunn
o Diabetic (2-13%) pupil, large number of cotton wool spots, widespread capillary
o Blood dyscrasia –hyperviscosity non perfusion
• Clinical Findings: (either BRVO or CRVO)
o Relative loss of the whole or part of the visual field SAMPLE CASE:
o Dilated, tortuous veins, flame shaped • 66/M, Hypertensive -sudden decrease in vision, left eye
hemorrhages, dot/blot hemorrhages, retinal • No pain, no eye redness, no discharge, no h/o trauma
edema, cotton wool spots, optic disc edema • VA: OD -20/20; OS –20/100
• PMHx: + HTN
• SLE OU: normal
• DFE: OD –AV ratio 1:3
• DFE OS:
o Flame shaped hemorrhages
o Cotton wool spots
o Dull foveal reflex
Impression: Retinal Vein Occlusion

Hotdog and catsup fundus

BRANCH VEIN OCCLUSION


• AV crossing - (vein is compressed by thickened artery)

TRANS BY: VIRATA AND SANTOS Page 14 of 51


OPHTHA FINALS TRANS

RETINAL ARTERY OCCLUSION HYPERTENSIVE RETINOPATHY


• True ophthalmic emergency and blood flow needs to be • Vasoconstriction
restored within minutes or hours before permanent damage o Generalized or focal arteriolar narrowing
occurs to the retina • Leakage
• Embolization is common cause of obstruction (heart and o Abnormal vascular permeability
carotids) o Flame-shaped hemorrhage, retinal edema, hard
• Raised intraocular pressure exudates
• Types: • Arteriosclerosis
o CENTRAL retinal artery occlusion (CRAO) o Thickening of vessel wall
o BRANCH retinal artery occlusion (BRAO)
• Clinical Findings: GRADING OF HYPERTENSIVE RETINOPATHY
o Acute, painless, profound loss of vision • MODIFIED SCHEIE CLASSIFICATION
(permanent) GRADE 0 No changes
o Marcus Gunn Pupil (+RAPD) GRADE 1 Barely detectable
o Pale retina GRADE 2 Obvious arterial narrowing with focal
▪ Diminished retinal blood circulation irregularities
o Cherry red spot (fovea) GRADE 3 Arterial narrowing, flame-shaped
▪ Orange reflex from choroidal vessels hemorrhages, cotton wool spots, hard
▪ Thin retinal layers at the macula thicken exudates
(retinal edema) GRADE 4 (+) Optic disc swelling
o May lead to optic disc atrophy after a few weeks

Cherry red spot Grade 4 (arrows) Flame-shaped hemorrhages


o Severe altitudinal visual field defect
o White ischemic/pale retina supplied by HYPERTENSIVE RETINOPATHY IN PREGNANCY
obstructed artery
• Seen usually in Eclampsia
• Symptoms:
o Scotoma, Dimming of vision, Diplopia,
Photopsias
• Findings:
o Retinal vascular constrictions & occlusions
o Progression of diabetic retinopathy
o Serous retinal detachment

RETINOPATHY IN BLOOD DYSCRACIA


• Ischemia
White ischemic pale retina o Sickle cell diseases
o Anemia/leukemia
TREATMENT o Hyperviscosity states
• Dislodging emboli
o Golden period 24 hours
o 90 minutes: irreversible damage to sensory
retina

MANAGEMENT
• Goal: Dislodge the emboli
o Patient must lie flat
o Firm ocular massage intermittently for 15 minutes
o Lower IOP, Increase blood flow, dislodge emboli
• I.V. acetazolamide
• 95% O2 inhalation+5% CO2 (brown bag breathing)
• Diagnostics are important!
o Carotid studies
o ECG, 2D-Echo

TRANS BY: VIRATA AND SANTOS Page 15 of 51


OPHTHA FINALS TRANS

OTHER RETINAL VASCULAR LESIONS


1. Retinal Telangiectasia • Drusen
• Rare, idiopathic, congenital o Earliest clinically
• Vascular tortuosity, exudation, leakage detectable feature
2. Coatsʼ Disease o Yellow excrescences
• Unilateral, idiopathic, telangiectatic and beneath the RPE
aneurysmal retinal vessels, intraretinal and o All eventually will have a
subretinal exudates drusen but not all will
develop AMD
• Adolescent boys
Drusen
3. Eales Disease
• Peripheral retinal vascular disorder (sheathing) TYPES OF DRUSEN
causing recurrent vitreous hemorrhage • Hard drusen- delineated border
• Bilateral, young men • Soft drusen- borders are not delineated and have increased
risk for AMD
1 2 PATHOGENESIS
• Deposition of abnormal material (drusen) in Bruch’ s
membrane which in turns damage the RPE

DRY VS WET AMD


DRY AMD
• Non exudative (geographical atrophy)
• Most common
3 • Slowly progressive atrophy of the RPE and photoreceptors
• Spectrum of findings:
o Drusen
o RPE atrophy
o Functional vision loss
o RPE pigment changes
• Sharply circumscribed circular areas of RPE atrophy
• Late stages → larger choroidal vessels seen
• When bilateral, lesions symmetrical
• Supportive treatment: Vitamins

MACULOPATHIES
SYMPTOMS OF MACULAR DISEASE
• Central blurring of vision or scotoma
• Metamorphopsia- distortion of image shape
• Micropsia- decrease in image size
• Macropsia- increase in image size

DRY AMD- geographical atrophy

• Question: Is drusen the same as the floaters?


• Answer: No. Floaters are caused by vitreous
condensations. Drusens are located below the retina. They
Central blurring of vision or scotoma seen in Age-related don’t float. They are deposited underneath the RPE.
Macular Degeneration
WET AMD
Scotoma= term used to define any area in the visual field that • Exudative type
is decreased in sensitivity compared to the surrounding areas. • Loss of central vision in days
Since it is located in the macula, it is called a CENTRAL • Two features:
SCOTOMA o RPE detachment
▪ Sharply circumscribed dome shaped
AGE-RELATED MACULAR DEGENERATION elevation of the retina
• Visual loss + Retinal Pigment Epithelium (RPE) ▪ Can develop a tear in the RPE
atrophy/subretinal neovascularization o Choroidal neovascularization
• 50 years and above ▪ Proliferation of fibrovascular tissue from
• Leading cause of permanent blindness in the elderly choriocapillaries
• Bilateral, Progressive ▪ Grey green or pinkish yellow slightly elevated
subretinal lesion – neovascular membrane

TRANS BY: VIRATA AND SANTOS Page 16 of 51


OPHTHA FINALS TRANS

• Fluorescein Angiography

(L)Proliferation of fibrovascular tissue(R)Neovascular Smoke Stacked pattern Ink blot Pattern


membrane
 Visual Prognosis:
• Treatment: o 80% - spontaneous resolution in 1-6 months
o Laser treatment- targets neovascularization to o 20% - may last 6 -12 months
prevent its growth. A Gold Standard treatment o Minority - permanent impairment of visual function
o Anti – VEGF therapy- any vasculopathies and o 40-50% - recurrences
inflammation of the eye • Management
o For acute, we don’t treat right away because 80%
resolves spontaneously.
o Advise the patients to wait 3-6 months. Advise
treatment if it lasts 6 months or sometimes 4 months
pwede na
o Laser → hasten symptomatic relief

Before Laser After Laser

CYSTOID MACULAR EDEMA


 Etiology
o With retinal vascular leakage
CENTRAL SEROUS RETINOPATHY ➢ Diabetic retinopathy, Branch Retinal Vein
• Unilateral, localized detachment of the sensory retina Occlusion, uveitis, telangiectasias
o Without
at the macula
➢ Retinitis pigmentosa, early macular hole
• Idiopathic, sporadic, self-limiting  Fluid between outer plexiform and inner nuclear layers
• Young-middle aged males  Fluid filled microcysts→ coalesce → lamellar hole
• Risk factors: Type A personality (CEO, OC med  Clinical Findings:
students), steroid users o Loss of foveal
• Dysfunctional RPE: Due to failure in the pumping depression- Because
mechanism of the RPE. If the pumping mechanism is there is thickening of
dysfunctional, fluid accumulates underneath the retina the retinal due to the
causing retinal detachment fluid accumulation
• Important features o Thickening of retina
o Shallow, round/oval elevation: Like a blister on o Multiple cystoid areas
the center of the macula  Fluorescein Angiography
o Borders outlined with glistening reflex o “Flower-petal pattern”

Flower-petal pattern
Oval elevation with glistening reflex

TRANS BY: VIRATA AND SANTOS Page 17 of 51


OPHTHA FINALS TRANS

 Treatment: STAGE IV
o Laser → Vascular cases  “With posterior vitreous detachment”
o Systemic CAI/ Steroids → Uveitis/inflammation

MACULAR HOLE
 Most commonly in women 6th-8th decades
 Etiology
o Idiopathic
o Severe myopia
o Trauma
o Solar retinopathy
▪ Small,
circumscribed
lamellar hole
or cyst
▪ 2 weeks post exposure to sun IV
▪ Cause by phototoxicity
 Fovea MYOPIC MACULOPATHY
o Thin  Productive years of young adulthood
o Avascular in the center  Progressive elongation of the globe
o Vitreous is very adherent to the fovea  Degenerative changes
 Focal contraction of the perifoveal vitreous cortex  Clinical findings:
 Well delineated, reddish round patch at the center of the o Islands of chorioretinal
macula (fovea). Sometimes described as pepperoni-like atrophy
lesion. o Atrophy of RPE and
 Treatment choriocapillaries
o Surgical o Exposure of of large
choroidal vessels
STAGE 1 o White sclerae
 “Impending Hole” o Lacquer cracks
 Progressive loss of the foveal depression
 I A - with appearance of initially a yellow spot at the fovea
 I B - enlarging to a yellow ring
EPIRETINAL MEMBRANES
 Idiopathic
o Posterior vitreous detachment
 Or caused by:
o Vascular occlusions, uveitis, trauma, intraocular
surgery, retinal breaks
 M=F, over 50 years of age
 Bilateral in 20%
 Metamorphopsia

CHOROIDAL FOLDS
IA IB  Etiology
o Idiopathic
STAGE II o Orbital diseases
 “Foveal Hole” o Choroidal tumor
 Centric or Paracentric o Posterior scleritis
o Ocular hypotony- Too much aqueous humor
STAGE III coming out of the trabeculectomy site can cause
 “Full thickness hole” severe decrease IOP causing hypotony.
 350-500um  Grooves and striae at the posterior pole

II-III

TRANS BY: VIRATA AND SANTOS Page 18 of 51


OPHTHA FINALS TRANS

TOXIC MACULOPATHIES ANGULAR BLEPHARITIS


 Post-eye maculopathy. There is pallor around the fovea • Etiology:
(atrophy). o Moraxella (Gram Negative Diplococci)
 Antimalarial drugs (CHOLOROQUINE AND • Signs:
HYDROCHLOROQUINE) o Lid angle maceration
o Malaria, rheumatoid, dermatologic o Conjunctivitis
o Concentrates in melanin containing structures
(RPE, choroid)
 Risk of toxicity → dose related
o 300g (250mg/day X 3 years)
o So if there is a patient taking these drugs monitor
for 3 months. Because it is dose related, follow-up
regularly.
 Cumulative dose < 100g
 <1 year
 Hydroxychloroquine better than chloroquine
 Stages: Lid angle maceration – moist, erythematous
o Premaculopathy
• Treatment:
o Established maculopathy
o Antibiotic eye ointment (in lateral canthus and
o Bull’s eye
conjunctiva) as needed
o Severe
o End stage
CHRONIC BLEPHAROCONJUCTIVITIS
• Scarring of the eyelid
• Etiology:
o Staphylococcal (Gram positive cocci in clusters)
• Signs:
o Discharge/Crusting/Ulceration on lid margins and
eyelashes “collarettes”
o Mild Conjunctivitis
o Redness and lid swelling
• Treatment:
o Lid scrub regiment with antibiotic (+/- steriods) ointment

Bull’s eye appearance

EXTERNAL EYE DISEASES


BLEPHARITIS
• Etiology:
o Staphylococcal (Gram positive cocci in cluster) –
most common cause
o Seborrheic
● Signs: HORDEOLUM
o Discharge/Crusting/Ulceration on lid margins and EXTERNAL HORDEOLUM
eyelashes “collarettes, scruffs” • Aka “stye” “kuliti”
o Collarettes – ulceration of hair follicles • Etiology
o Mild Conjunctivitis o Inflammation of hair follicles and its glands
o Redness and lid swelling • Signs
(+) inflammation at the periorbital areas o Discharge/Crusting/Ulceration on lid margins and
• Treatment: eyelashes “collarettes”
o Lid scrub regiment with antibiotic (+/- steriods) o May have Conjunctivitis
ointment • Treatment
o Lid scrub regiment with antibiotic (+/- steriods)
ointment

TRANS BY: VIRATA AND SANTOS Page 19 of 51


OPHTHA FINALS TRANS

INTERNAL HORDEOLUM CONJUNCTIVITIS


• Aka Meibomianitis/Meibomitis Papillae:
• Etiology: • Fibrovascular mounds with central vascular tuft (Red)
o Inflammation of the meibomian glands (found on • Nonspecific, different causes
your tarsus, posterior part of the lid)
• Signs:
o Eyelid Mass
o Eye redness
• Treatment:
o Lid scrub regiment with antibiotic- steroid ointment
o Warm compress

Papillae
FOLLICLES
• Translucent, avascular mounds of lymphoid tissue
(White)
CHALAZION • More specific etiologies
• Etiology:
o Chronic lipogranulomatous inflammation from an
obstructed meibomian gland
o Usually due to unresolved internal hordeolum
• Signs:
o Eyelid non-tender mobile mass
o (+) granulation
o Chronic – no inflammation, no redness and
swelling
• Treatment:
o Incision and curettage Follicles
o May try medical treatment first (antibiotic-steroid
ointment, lid scrub, warm compress) VERNAL KERATOCONJUNCTIVITIS
• A form of chronic papillary conjunctivitis
• Etiology:
o Type 1 and type 4 Hypersensitivity Reaction
o Usually in young males with history of atopy
• Signs:
o Eyelid redness and pruritus (good hallmark for
allergic disease)

OPHTHALMIA NEONATORUM
• “Neonatal Conjunctivitis”
• Etiology:
• Chlamydia (most common), staphylococcal, gonorrhea
and herpes
• Gonococcal-acute (1st 2-3 days), severely purulent,
potentially blinding
• Chlamydia- chronic (1st 2 weeks)
• Signs:
o Eye redness COBBLESTONE PAPILLAE
o No follicles-develops after a few weeks
• Treatment:
o Erythromycin for chlamydia, ceftriaxone for
gonorrhea

SHIELD ULCER
• Treatment:
o Mast cell stabilizers-> Steroids

TRANS BY: VIRATA AND SANTOS Page 20 of 51


OPHTHA FINALS TRANS

TUBERCULOUS CONJUNCTIVITIS VIRAL BACTERIAL ALLERGIC


• A form of chronic papillary conjunctivitis Laterality Bilateral Bilateral or Bilateral
• Etiology or Unilateral
o Granulomatous reaction to TB infection initially
• Signs unilateral
o Conjunctival granulosa (important to rule out Signs and Red/ pink Red/ pink Red/ pink
TB) Symptoms eye(s) eye(s) eye(s)
o Eye redness and pruritus Eye Eye Discharge Eye
• Treatment Discharge Itchiness Discharge
o Systemic anti-TB medication Itchiness Lid edema Itchiness
Lid Lid edema
edema
VIRAL CONJUNCTIVIS Discharge Serous Purulent Serous
• “TRUE Sore Eyes” (watery) (watery,
• Etiology: ropy,
o Adenovirus type 8, 19, 27, 37- more common stringy
“Epidemic Keratoconjunctivitis” Managemen Symptom G/S Culture Antihistamin
o Enterovirus 70- presents as Acute hemorrhagic t a Antibiotic eye e (oral
Conjunctivits tic drops: or topical)
• Signs: Cold (Aminoglycosid Mast cell
o Diffuse eye redness and serous/watery compress e/ stabilizers
discharge, foreign body sensation Artificial Fluoroquinolone Artificial tear
• Treatment: tears )
o Conservative (cold compress, lubricant) Sign Follicle ± Papillae Papillae
o STRICT HAND HYGIENE papillae
History Recent Recent Asthma,
exposure, exposure, eye Dermatitis,
eye rubbing, URTI postnasal
rubbing, drip
URTI
Organisms Adenoviru Staphylococcus None
s sp.
(serotype Streptococcus
s sp
3 & 7)

PINGUECULA
• Degenerative lesion of the bulbar conjunctiva
• Etiology:
o Secondary to chronic sunlight exposure
• Signs:
o Yellow white amorphous lesion interpalpebral
zone
• Treatment
o Observation and supportive treatment
PHLYCTENULAR CONJUNCTIVIS
• Aka “phylctenulosis
• Phlycten- focal, nodular, vascularized infiltrate of PMNS
and lymphocyte with central necrosis
• Etiology:
o Type IV hypersensitivity usually to TB (mc in
philippines) and Staphylococcus
• Treatment
o Topical steroids, treat primary infection

PTERYGIUM
• Etiology:
o Actinic degeneration of the conjuctiva and
cornea secondary to chronic sunlight exposure
o May be preceded by a pinguecula
• Signs:
o Triangular wing-shaped fibrovascular mass
gradually encroaching the cornea
• Treatment:
o Pterygium excision with conjunctival grafting

TRANS BY: VIRATA AND SANTOS Page 21 of 51


OPHTHA FINALS TRANS

o Includes topical antivirals (if epithelial), topical


antibiotics and topical steroids (if
stromal/endothelial)

KERATITIS
BACTERIAL KERATITIS
• “Aka Infectious Keratitis, Corneal Ulcer
• Etiology:
o Most common: Pseudomonas, Moraxella,
Streptococcus
• Signs:
o Central corneal opacity
o Eye redness and foreign body sensation
o Feathery border for fungal CORNEAL SCAR
• Treatment: • Etiology:
o Gram stain and culture o Trauma
o Topical antibiotics (broad spectrum-> targeted) o Infection
• Signs:
o Central corneal opacity
o NO eye redness and foreign body sensation
• Treatment:
o May try steroids, usually requires corneal
transplant if central in location
o If not in visual axis: observe
o If within visual axis(visual impairment): corneal
transplant

SUBCONJUNCTIVAL HEMORRHAGE
• Etiology
o Trauma
o Spontaneous rupture of conjunctival vessels- Due to
Hypertension and Valsalva’s manuever
Gonococcal • Signs:
Conjunctivitis o “Bright red” eye redness
o No eye pain, no foreign body sensation
o May occur in px with LEUKEMIA or BLOOD dyscrasia
HERPES KERATITIS • Treatment:
• Aka Infectious keratitis, Corneal ulcer o Observation
• Etiology: o Supportive Management→ Warm Compress
o HSV1, HSV2
• Recurrent, may reactivate later especially if
immunocompromised
• Signs:
o Usually dendritic opacity, nummular opacity
o Recurrent episodes
o Decrease corneal sensation
• Treatment:
o Dependent on layer of involvement

TRANS BY: VIRATA AND SANTOS Page 22 of 51


OPHTHA FINALS TRANS

TRAUMATIC HYPHEMA with systemic


• Etiology: collagen
o Trauma(blunt or penetrating) vascular disease
• Signs: (majority
o Blood in anterior chamber with rheumatoid
• Treatment: arthritis)
o May observe initially for resolution – can drain itself Management None Refer to an
o Surgical washout if persistent or with development of Artificial tears ophthalmologist
glaucoma Mild topical • Oral NSAIDs
steroids • Oral steroids
• NSAIDs +
steroids
• Subconjunctival
steroid injection
Natural History Benign, self- More aggressive
limited Needs medication
Usually resolves
in 1-2 weeks
CORNEAL ABRASION
ARCUS SENILIS • Corneal lesions are lesions at the inferior cornea.
• Etiology:
o Age related changes in metabolism
• Signs:
o White ring at the limbal area

THE RED EYE


DYSFUNCTIONAL TEAR SYNDROME • There is denudation
• Previously known as Dry Eye Syndrome • Common among who don’t wear contact lenses
• Signs and Symptoms: properly or slept with contacts on.
o Tearing • Pain is the characteristic of corneal abrasion and this is the
o Eye discomfort (sometimes described as mild most painful.
pain) • Fluorescein dye stains where epithelium isn’t intact→
o Itchiness Helps identify areas of abrasion
o Foreign body sensation • Signs:
• Management: o Eye redness
o Artificial tears o Abrasion more visible with dye stain
• Symptoms:
o Steroids if with severe dry eye, until inflammation
o Mild to severe pain
is controlled (not for long term use, may cause
o Tearing
glaucoma) o Difficulty opening eyes/photophobia
o Avoidance/control of triggers (direct fan/ aircon, • Management:
screen time, etc) o Remove corneal FB if present
o Advise and educate the patient o Prophylactic antibiotics, lubricants
o Patch eye vs bandage contact lens as necessary
EPISCLERITIS
• is swelling of the DEEP blood vessel plexus in the surface
of the sclera or beneath the conjunctiva.

EPISCLERITIS SCLERITIS
Inflammation of Inflammation of
the episclera the
sclera
Pain Mild to none More severe
Reaction to Vessels blanch Vessels are not
Phenylephrine (turn white) affected
drops (remain red)
Demographic Young adult Older adult or
(young or old)
patients
TRANS BY: VIRATA AND SANTOS Page 23 of 51
OPHTHA FINALS TRANS

CHEMICAL CORNEAL INJURY ACUTE ANGLE CLOSURE CRISIS


• TRUE OCULAR EMERGENCY • Caused by Pupillary Block:
• Alkali causes more damage than acid injury o fluid is trapped behind iris, can’t go through
• If surely chemical injury, TREAT FIRST before taking normal pathway through pupil to trabecular
history meshwork
• Management: o high intraocular pressure (30 - 70! mm Hg)
o Copious irrigation,
▪ With balanced salt solution, normal saline, LR
(lactated ringer)
▪ Must check if surface pH (7.0-7.4)
o Topical anesthetic for pain
o Wash and sweep fornices
o Ophtha referral a must
o Prophylactic antibiotic, +/- steroid
o Prognosis depends on severity
▪ Corneal defect, stromal haze, limbal ischemia
▪ Red eye might be better than white eye
▪ White eye could mean ischemic eye

• Signs:
o Eye redness
o Sudden Blurred vision
o Corneal edema
o Headache (periorbital)/nausea
o mid dilated pupil
o Glaucomflecken
ACID CORNEAL INJURY o gray-white epithelial and anterior cortical lens
• Less severe damage than alkali burn opacities that occur following an episode of
• Acid denatures proteins, causes fibrosis markedly elevated IOP, as in acute angle-closure
• Barrier forms and limits further tissue penetration o Shallow anterior chamber
• Management:
o indentation gonioscopy to open angles
o Give topical and possibly oral IOP lowering meds
o definitive management: iridectomy

ALAKALI CORNEAL INJURY


• Can cause more damage
• Alkali causes
o Cellular saponification
o Destruction of collagen fibers ORBITAL AND LACRIMAL DISEASES
o Penetration of corneal stroma INFECTIOUS
• As long as chemical still there destruction can continue. Preseptal Cellulitis
CILIARY FLUSH • Inflammation of the tissues localized anterior to the orbital
septum
• A.k.a circumlimbal swelling. Swelling does not come from
periphery as in the case of episcleritis. It comes from the • Common cause: skin infections (Staphylococcus,
limbus moving backward streptococcus)

SIGNS
• LID EDEMA, ERYTHEMA
• Normal visual acuity
• Absence of proptosis
• Normal conjunctiva
• Normal pupillary reaction
• Normal ocular movements
• Normal color vision

TRANS BY: VIRATA AND SANTOS Page 24 of 51


OPHTHA FINALS TRANS

TREATMENT • This complex web of veins contains no valves; blood can


• Topical and orbital antibiotics (usually for gram positive flow in any direction depending on the prevailing pressure
organisms) gradients
• NO need to admit
Etiology
• Staphylococcus aureus – approximately 70% of all
infections

Clinical Presentation
• Fever, chills, nausea and vomiting, headache
• Exophthalmos
• May have bilateral orbital signs
• Proptosis
• Decreased vision
• Absent pupillary reflexes
• Decreased corneal sensation secondary to CN V
Orbital Cellulitis • CN VI typically affected FIRST (Lateral gaze palsy) and
• Inflammation of soft tissues POSTERIOR to the orbital the most affected cranial nerve because CN VI is not
septum protected by any bones in the orbit.
SIGNS • CNS involvement: altered mental status, meningeal signs,
• Lid edema and erythema lethargy
• Proptosis
• Decreased visual acuity Imaging
• Swollen conjunctiva(chemosis) • MRI with MR Venogram – study of choice!
• Limitation of EOMs
• (+) Relative Afferent Pupillary Defect (RAPD) Management
In orbital cellulitis, grossly similar with preseptal cellulitis • IV antibiotics
pero kapag inopen mo yung mata, hindi lang yung skin but • Consider heparin in rapidly decompensating
also all structures in the orbit are involved.
INFLAMMATORY
WORK-UP
Thyroid Related Eye Disease (TRED)
• Orbital CT-scan
• Autoimmune inflammatory disorder
• Diagnosis is based on the presence of (2 of 3):
TREATMENT
1. Immune related thyroid dysfunction
• ADMIT the patient (Thyrotoxicosis/ Grave’s disease)
• Intravenous antibiotic 2. Orbital signs
• Monitor signs daily and note any improvement/worsening 3. Radiographic evidence
• May be seen in the euthyroid state
• Most common cause of
o Lid retraction – most common feature
o Unilateral and bilateral ptosis

Cavernous Sinus Thrombosis (CST)


• A late of an infection of the central face or paranasal sinuses
o May develop from orbital cellulitis (via superior
ophthalmic vein)
• Important to note that once the infection progressed in the
cavernous sinus, manifestation is usually bilateral orbital
signs and infection, is in close proximity with your brain,
hence, increase risk for meningitis.

Pathophysiology Pathophysiology
• Cavernous sinuses receive venous blood from the facial • Orbital fibroblasts cause an upregulation and production of
veins (via the superior and inferior ophthalmic veins) as well glycosaminoglycans and adipogenesis in the orbit
as the sphenoid and middle cerebral veins • Cross-reaction happens between your orbital fibroblast
reacting to thyroid hormones causing fat accumulation and
glycosaminoglycan deposition leading to proptosis.

TRANS BY: VIRATA AND SANTOS Page 25 of 51


OPHTHA FINALS TRANS

Management
• Corticosteroids – oral or IV
Treatment goals • Orbital biopsy – when not responsive to steroid treatment
• Control and reduce risk factors- Control the hyperthyroid
NEOPLASTIC
state.
• Restore euthyroid state Capillary Hemangioma
• Minimize effects of active phase • Most common PEDIATRIC benign tumor
o Oral steroids to reduce inflammation • Present at birth or a few weeks after
• Treat residual effects • 75% of lesions resolve after 4-5 years.
o Surgical management once disease is stable/inactive • Management:
(orbital decompression, muscle surgery, eyelid o Observe small lesions
surgery) o Refract to best corrected VA; If with amblyopia, treat
o Medical: systemic or intralesional steroids; systemic
Nonspecific Orbital Inflammation (NSOI) propranolol
o Surgical: If refractory to treatment
• Previously called “orbital pseudotumor” or “idiopathic
orbital inflammatory syndrome”
• A benign inflammatory process of the orbit characterized
by a polymorphous lymphoid infiltrate with varying degrees
of fibrosis, without a known local or systemic cause
• A diagnosis of exclusion

Cavernous Hemangioma
• Most common benign tumor in ADULTS
• Slow, progressive proptosis that becomes worse in
pregnancy
Pathophysiology • Well-encapsulated, red lesion
• An immune-mediated process because it is often • Management: Excision if with visual compromise
associated with systemic immunologic disorders
including Crohn disease, systemic lupus erythematosus,
rheumatoid arthritis, diabetes mellitus, myasthenia gravis,
and ankylosing spondylitis

Clinical Presentation
• 5 orbital locations (in order of frequency):
o EOMs (myositis),
o lacrimal gland (dacryoadenitis),
o anterior orbit (scleritis)
Optic Nerve Glioma
o orbital apex
o diffuse inflammation • A slow-growing tumor that typically affects children
• Deep-rooted, boring pain; EOM restriction, proptosis, • Gradual painless, unilateral axial proptosis
conjunctival inflammation, chemosis, eyelid erythema and • Vision loss due to optic nerve involvement.
soft-tissue swelling • Associated with neurofibromatosis
• Management:
Imaging o Depending on tumor growth and extent of optic nerve
• Ct-scan and chiasmal involvement.
• MRI o Chemotherapy or surgery

TRANS BY: VIRATA AND SANTOS Page 26 of 51


OPHTHA FINALS TRANS

Optic Nerve Sheath Meningioma


• Commonly affects women in the 3rd to 4th decade
• Rarely: axial proptosis with preserved vision
• Management:
Rhabdomyosarcoma
o Depending on tumor growth and extent of involvement
o Fractionated radiotherapy • Most common primary ORBITAL malignancy
• commonly seen in ages 8-10 years
• High index of suspicion in children with a rapid onset of
unilateral proptosis.
• Palpable mass in the superonasal orbit
• Immediate biopsy/imaging required
• Management:
o Chemotherapy + Radiotherapy

• ON-SM is for adult: ONG is for pedia


• ON-SM is tracking: ONG is kinking
• ON-SM is radiotherapy: ONG is chemotherapy LACRIMAL SYSTEM
Physiology: Lacrimal Pump
Retinoblastoma • Dependent on lid closure and lid opening
• Most common primary INTRAOCULAR malignancy • Lid CLOSURE: formation of negative pressure through
• It is a life threatening condition! opening of the sac
• Commonly presents before age 3 • Lid OPENING: formation of positive pressure as lacrimal
sac collapses
• Mutational loss of BOTH copies of the RB gene in
chromosome 13q14 (long arm of chromosome 13) Congenital Nasolacrimal Duct Obstruction
o Two hit-hypothesis • Usually caused by a membranous block of the valve of
• Most common presenting signs are leukocoria and Hasner
strabismus • Most obstructions open spontaneously within 4-6 weeks
• Most cases are sporadic and unilateral after birth
• If it is familial, or bilateral, you expect a worse prognosis. • Approximately 90% of congenital NLDO’s resolve in the
• Management 1st year of life
o Enucleation • Management:
o Chemotherapy o Observe if <1 year old
o Radiation o Referral to ophthalmologist if persistent >1-year-old
o Cryotherapy
If detected earlier, the eye can be preserve that’s why Acquired Nasolacrimal Duct Obstruction
it is important na madetect agad yung leukocoria or
• 2 groups: Lacrimation vs epiphora
cat’s eye reflex.
o Lacrimation – hypersecretion of tears
• Retinoblastoma is one of the differential diagnosis if you
o Epiphora– impairment of drainage,
have a negative ROR.
pathophysiology of acquired NLDO
Etiology
• Involutional stenosis - most common
• Inflammatory disease (recurrent dacrocystitis)

TRANS BY: VIRATA AND SANTOS Page 27 of 51


OPHTHA FINALS TRANS

Management IMAGING
• Intubation & stenting • B scan (ocular ultrasound) → rule out open globe
• Dacryocystorhinostomy (DCR) – Definitive Treatment: • CT scan is for fractures vs. MRI is contraindicated in
create an anastomosis between the lacrimal sac nasal suspected metallic foreign bodies
cavity through a bony ostium
Indications ORBITAL FRACTURE
• Recurrent dacryocystitis • Blowout fracture
• Chronic mucoid reflux o Fracture of one or more orbital bone
• Painful distension of lacrimal sac o Force from an object that hits the orbital bones
• Bothersome epiphora (usually the eyebrow or frontal bone and upper
cheek bone or maxillary bone is transmitted to the
OCULAR AND ORBITAL INJURIES bones
• Most common among children and young adults, usually o Usually from a blunt object larger than the orbital
Males aperture or the eye socket (e.g Tennis ball,
basketball and fist)
HISTORY
• Onset of injury
• Mechanism of injury

• The onset and mechanism of injury are two of the most


essential information that you need to elicit in an ophtha
patient because they have something to do with the
prognosis

OPTHALMOLOGIC EXAM ORBITAL FRACTURE SIGNS


• Visual acuity- useful in prognostication of trauma • Proptosis
patients Kapag may Nakita kayong Test VA yun agad o Abnormal protrusion of the globe
sagot! • Enophthalmos
• Pupils o Posterior displacement of the globe within the orbit
o Check for direct and consensual pupillary • Hypoglobus
response o Downward displacement of the globe
o Compare the reaction of two pupils • Diplopia
o Check for nerve defect (RAPD) o Doubling of vision
• Extraocular muscle motility and alignment • Infra-orbital nerve (CN V2) involvement
o Check for diplopia- limitation or restriction on o Observed as cheek hypoesthesia
movement of EOM’s; should be binocular. Both o Very specific to an inferior wall fracture
eyes should be affected
• Intraocular pressure – AVOID DIGITAL PALPATION if
suspecting open globe injuries
o May also indicate that there is a possible bleeding
inside the eye
• External/gross examination
o Check the status of cornea, anterior chamber and
iris
• Slit lamp exam Left pic: Proptosis and Hypoglobus Right pic:
o Examination of Iris Enophthalmos
• Funduscopic exam
o Examination of the posterior structure of the eye
• Confrontation visual field examination’
o Not urgent; last part of the eye examination
• Test for infra-orbital skin hypoesthesia
o Involvement of the infra-orbital nerve
o Consider orbital floor fracture
• Fluorescein dye test
o Yellow-Orange in color (Fluorescein dye)
o Dye uptake – corneal abrasion/ laceration
o Seidel Test – aqueous leakage/ perforation

Exophthalmos

Corneal Abrasion Siedel Test

TRANS BY: VIRATA AND SANTOS Page 28 of 51


OPHTHA FINALS TRANS

ORBITAL FRACTURE SURGERY • Bulbar Conjunctiva:


• Indications for surgery o Foreign body
o Persistent diplopia within 30° of the primary ▪ Evert the upper and lower eyelids to
position of gaze with evidence of soft tissue check for foreign body
entrapment or large fracture ▪ Use a fine gauge needle or a spud to
o Persistent oculocardiac reflex (trapdoor fracture remove the foreign body during slit lamp
in children) – exam
o Early enophthalmos or hypoglobus ❖ Do not use a cotton-tipped
• Do not do surgery right away because we expect the eye applicator because it rubs off a
to be swollen right after injury. There is a possible large area of epithelium, often
contraction of the orbital contents. without removing the foreign
• May observe for 1-2 weeks for resolution of diplopia body.
❖ Prescribe antibiotic eye drops or
• Longer delays decrease the likelihood of successful repair
ointment after removal of a
due to progressive scarring
foreign body
▪ Examine the wound to rule out infection
OCULAR INJURIES
until it is healed.
Birmingham Eye Trauma Terminology System o Wound → clean with NSS, oral antibiotics
• Control ocular and orbital pressure to prevent intraocular ▪ Check for any lid margin involvement
extrusion in an open globe (Full-thickness laceration and (laceration or avulsion) → refer to an
there may be prolapse of iris, lens, etc.) ophthalmologist
• BIRINGHAM EYE TRAUMA TERMINOLOGY SYSTEM ▪ Check for medial canthus involvement
(BETTS) →refer to an ophthalmologist
o Closed globe – lamellar laceration or partial- o Penetration
thickness laceration o Perforation
o Penetrating laceration – Entrance only
o Perforating laceration – Entrance and exit

APPROACH TO OCULAR INJURIES


Assess vital signs, ABCs (airway, breathing circulation)

Determine the Visual Acuity (VA)


• To document the baseline
• For prognostication of visual outcome Foreign body under the lid in scratching the corneal surface
• To measure VA, one may use:
o EDTRS- standard tool in measuring VA
o Snellen
o Rosenbaum or Jaeger Near Vision Chart

Test pupillary light reflex


• Check for RAPD

Gross examination (external to internal)


• Adnexae and Ocular Surface Left: Right upper mid margin avulsion. Right: Extramarginal upper
o Check for abrasion, laceration, avulsion lid laceration from blunt trauma in an infant, Such lacerations tend
o Evert eyelids for foreign body, remove if possible to follow relaxed skin tension lines.
(under the slit lamp)
o Check for any lid margin involvement or possible Anterior segment
lacrimal apparatus transection (Indications for • Anterior chamber
referral to an ophthalmologist for repair) o Foreign body
o Check for aqueous leaks (Siedel test) o Hyphema – blood in the anterior chamber
o Penetrating or perforating injury (presents as
shallow anterior chamber)
• Pupil
o Size
▪ Pupillary light reflex
▪ (+) RAPD, soft and unformed eye, and
vitreous hemorrhage suggests globe
rupture
o Shape
▪ Corectopia (irregularly shaped pupil) –
may be due to iris prolapse
Siedel Test ▪ Traumatic mydriasis

• Ocular Surface
o Foreign body (usuallly in blast injuries, hammering
of metal-tometal)
TRANS BY: VIRATA AND SANTOS Page 29 of 51
OPHTHA FINALS TRANS

Shaken Baby Syndrome


Traumatic Mydriasis Corectopia Subretinal, Intraretinal and Pre-retinal Hemorrhages

o Surgical evacuation of hyphema may be indicated Posterior segment (cont..)


if IOP remains elevated: • Optic Nerve
▪ >35mmHg for 7 days o Optic nerve avulsion
▪ >50 mmHg for 5 days o Traumatic optic neuropathy (edema and atrophy)
o However, there is a risk of rebleeding.
o Pupil dilation may increase the risk of rebleeding
and may be deferred until the hyphema has
resolved by spontaneous absorption.
o Frequently examine the eye for secondary
bleeding, glaucoma, or corneal blood staining from
iron pigment.
• Slit Lamp Exam: Iris
o Iridodialysis- Iris detachment
o Angle recession – separation of longitudinal and
circular iris muscles (Done by an ophthalmologist Optic Nerve avulsion Traumatic Optic Neuropathy
through gonioscopy)
• Lens LAMELLAR LACERATIONS
o Traumatic cataract -> refer to an ophthalmologist
o Lens subluxation -> refer to an ophthalmologist • Corneal abrasion
o Lens dislocation -> refer to an ophthalmologist o Instill tetracaine anesthetic drops, topical
antibiotics, bandage contact lens (if no infection is
suspected), or eye patch (not as commonly done
nowadays)
• Foreign body in cornea, conjunctiva, or sclera
o Remove the foreign body through fine forceps or
fine gauge needle

OCULAR AND ORBITAL INJURY: REMINDERS


• Never give a topical anesthetic to the patient for repeated
use after the injury (delays healing, masks further damage,
Left pic: Traumatic Iridodialysis Right pic: Lens can cause corneal ulcer and permanent corneal scar).
Subluxation • Never dilate a patient who has not undergone a neurologic
examination
Posterior segment • Never use non-sterile topical eye medications for open
• Vitreous globe injuries
o Vitreous hemorrhage • Never apply ocular or orbital pressure on a patient with
o Vitritis -> look for infection due to penetrating or open globe
perforating injury
• Retina OCULAR AND ORBITAL INJURY: GENERALIZATION
o Retinal detachment and tear • Inferotemporal limbus – most exposed to trauma
o Commotio retinae or Berlin’s edema • Superonasal limbus – most common site of globe
▪ Cherry red fundus against a whitish rupture (via contrecoup effect)
edematous retina • Blunt traumatic injuries have a worse prognosis due to the
increased incidence of retinal detachment and intraocular
tissue avulsion and herniation
• Any injury severe enough to cause intraocular hemorrhage
increases the risk of delayed secondary hemorrhage and
possible intractable glaucoma
• Corneal lacerations or perforating injuries must be repaired
to form a watertight closure
• The fellow eye is susceptible to sympathetic ophthalmia
whenever penetrating trauma occurs, especially if there has
Left pic: Commotio Retinae/ Berlins Edema Right pic: been damage to the uveal tissues
Retinal Tear o Sympathetic ophthalmia - bilateral,
granulomatous uveitis that occurs after trauma to
the eye

TRANS BY: VIRATA AND SANTOS Page 30 of 51


OPHTHA FINALS TRANS

THERMAL INJURY
• Thermal burns of the eyelids
o Topical antibiotics and sterile dressing
• Infrared irradiation
o Exposure from heat from sunlight, fire, radiator, or
warm sidewalk
o Can lead to Glassblower’s cataract
• Radiation (xray) and nuclear radiation -> cataract

Norms for Visual Acuity in Children


Cataract due to radiation Age (in months) Visual Acuity
• Ultraviolet irradiation- most common sa welders
o Usually due to exposure to electrical welding arc, 30-35 months (2-3 years old) 20/60 or better
electric sparks, suntanning bed, reflection from 36-47 months (3-4 years old) 20/50 or better
snow 48-59 months (4-5 years old) 20/40 or better
o Signs/Symptoms:
60-72 months (5-6 years old) 20/30 or better
▪ Eye pain, redness/ciliary flush,
photophobia, superficial keratitis
o Management Congenital glaucoma
▪ Fluorescein dye under cobalt blue light to • Another condition which presents as tearing in infants
identify SPKs (superficial puncate aside from congenital nasolacrimal duct obstruction
keratitis), lubricating gel or ointment, oral • Signs and Symptoms:
NSAIDs o Epiphora (excessive tearing)
o Enlarged eyes (Buphthalmos)
PEDIATRIC OPHTHALMOLOGIC EXAM o Photosensitivity / Photophobia
• Vision screening is recommended at least once in all o Corneal clouding
children aged 3-5 years.
OCULAR FIXATION
VISUAL ACUITY TEST • Ocular fixation should be evaluated routinely to test vision
• Eye Charts in an infant.
o Preliterate children:
▪ Picture charts (Lea or Allen) Age Response
▪ Matching charts (HOTV) 6 weeks old Some response to an
o Literate children: examiner’s face
▪ Letter charts (Snellen) 2 months old and older Child should fix and follow a
target
RED FLAG!! If at 3 months old, there is no binocular fix and
follow response → refer to an ophthalmologist

OCULAR ALIGNMENT
• To identify strabismus (eye misalignment), which may be
caused by
o Abnormal innervation of the EOMs
o Other pathology (ex: amblyopia, cataract,
retinoblastoma)
Remember: Any condition resulting to poor vision in one
eye can lead to misalignment!
• Test the corneal light reflex in all children.
• Do basic cover tests in children (>3years old) to identify
strabismus
Some kids cheat and peek through the occlude so you have to
make sure that the other eye is properly covered. Some of them
actually memorize the charts kaya right now mas preferred ang
projector chart where you can isolate one letter to discourage
kids from memorizing the lines.

TRANS BY: VIRATA AND SANTOS Page 31 of 51


OPHTHA FINALS TRANS

Remember!
Manifest= Tropia. Obvious. Kita mo agad
Latent= Phoria. Mukhang normal. Makikita mo lang pag
tinakpan mo yung mata biglang may deviation.

DEFINITION OF TERMS
• Angle Kappa- The main angle between the visual axis and
the central pupillary line. When the eye is fixing a light, if the
corneal reflection is centered on the pupil, the visual axis
and the central pupillary line coincide and the angle kappa
is zero.
• Conjugate movement: Movement of the eyes in the same
direction at the same time.
• Deviation: Magnitude of ocular misalignment, usually
measured in prism diopters but sometimes measured in
degrees.
• Comitant deviation: Deviation not significantly affected by
which eye is fixing or direction of gaze, typically a feature of
childhood (nonparetic) strabismus.
• Incomitant deviation: Deviation varies according to which
eye is fixing and direction of gaze, usually a feature of
recent onset extraocular muscle paresis and other types of
acquired strabismus.
• Primary deviation: Incomitant deviation measured with the
normal eye fixing.
• Secondary deviation: Incomitant deviation measured with
the affected eye fixing.
• Fusion: Formation of one image from the two images seen
simultaneously by the two eyes. Fusion has two aspects.
Hirschberg Test. You may use a penlight or an
o Motor fusion: Adjustments made by the brain in
ophthalmoscope. Ask the patient to fixate at a near target. A
innervation of extraocular muscles in order to bring
normal finding is the presence of the corneal light at the center
both eyes into bifoveal and torsional alignment.
of the pupil. An esotropic eye will present with a light reflex that
o Sensory fusion: Integration in the visual sensory
is temporal to the pupil. In exotropia, the light reflex is nasal to
areas of the brain of images seen with the two
the pupil. In Pseudostrabismus, minsan the broad nasal bridge
eyes into one picture.
or skin folds creates an illusion na merong esotropia pero pag
chineck mow ala naman. Pag ganito, advise the mom to observe
Types of Strabismus:
lang kasi as the child grows, mawawala din yan kapag
nagdevelop na yung facial bones
Heterotropia (paralytic or non-paralytic)
• Aka manifest deviation
STRABISMUS Types:
• Under normal binocular conditions, the image of the object
of regard falls simultaneously on the fovea of each eye, Exotropia Lateral Deviation, “wall-eyed”
(bifoveal fixation) and the vertical retinal meridians are both Esotropia Medial Deviation, “cross-eyed”
upright.
• Bifoveal fixation Hypertropia Upward deviation
o In normal binocular viewing condition Hypotropia Downward deviation
o Target object falls on the fovea of each eye
• Strabismus – any deviation from perfect ocular alignment
o The target object is not visualized simultaneously Pseudo Esotropia
by fovea of each eye (not bifoveal fixation) • Epicanthal folds give appearance of esotropia but
o Strabismus is not outgrown Hirschberg test is normal
o Types: • More common in Asians
▪ Manifest strabismus • Resolves as the nasal bridge and bone structure develops
(Heterotropia/Tropia) – present under with age
binocular viewing condition (or both eyes
are open) Cover Test
▪ Latent strabismus • Ask the patient to fixate on a distant target (as
(Heterophoria/Phoria) – present only opposed to Hirschberg test na NEAR TARGET)
when binocular vision has been • Cover the fixating eye, the deviated eye will then move
interrupted (or 1 eye is occluded) to fixate on the target
o Management: Treat ASAP to ensure best visual • The deviation can be quantified using prisms.
acuity and binocularity→ may lead to ambylopia if
not treated

TRANS BY: VIRATA AND SANTOS Page 32 of 51


OPHTHA FINALS TRANS

This test is performed after single cover testing as it is the most


dissociative of cover tests. It involves covering one eye and
holding the occluder for several seconds to suspend fusion, then
shifting the occluder to the other eye and rapidly alternating back
and forth without allowing the patient to become binocular and
being careful to always keep one eye occluded. The eye under
the occluder is observed as the occluder is removed and placed
over the fellow eye in order to determine the direction of
deviation. If there is an outward, or lateral, refixation in the
nasal to temporal direction- this represents an esodeviation.
If there is an inward, or medial, refixation in a temporal to
• Esotropia- OS left eye moves OUTWARD as the OD nasal direction – this represents an exodeviation. If there is a
right eye (initially fixating) is covered vertical refixation movement – this represents either a hyper or
• Exotropia- OS left eye moves INWARD as the OD hypo deviation. There can be both horizontal and vertical
right eye (initially fixating) is covered deviations present at the same time.
Heterophoria
AMBLYOPIA
• aka latent deviation
• Deviation corrected in the binocular state by the fusion • Also known as “Lazy Eye”
mechanism (the deviation is not seen when patient is using • Reduction of best-corrected visual acuity due to cortical
both eyes) suppression of sensory input from an eye that is receiving
• Normal Hirschberg test (symmetrical light reflexes) blurred or conflicting visual info, leading to disruption of the
normal development of visual pathways serving that eye.
• Most are asymptomatic
• Etiologies include:
• may present as asthenopia (eye strain, fatigue)
o Strabismus
• Hyperphopia- Tendency for one eye to deviate upward
o Anisometropia
• Orthophoria- The absence of any tendency of either eye to o Any ocular pathology that prevents proper light
deviate when fusion is suspended. This state is rarely seen transmission and transduction
clinically. A small phoria is normal
• Must be suspected if with the following:
Cover-Uncover Test
o Resistance to monocular occlusion
o Head tilt
o Nystagmus
• Leads to permanent vision loss if left untreated
• The presence of amblyopia always warrants referral to an
ophthalmologist.
• General Treatment of all types of amblyopia:
o Correct the underlying cause
o Occlusion therapy (patching) or atropine
cycloplegia (optical degradation therapy) of the
good eye

Always Patch the good eye! Para ma force yung bad eye na
In phoria, both eye are straight on primary gaze umayos. PUT ATROPINE ON THE GOOD EYE! Mag didilate
Look at the picture, Phoria naman to. Ang gumagalaw naman yung pupil ng good eye. This way, you induce BOV (ang effect
dito is yung eye UNDER the occlude. So paano mo makikita nito is 2 weeks na blurred vision) of the good eye so ganon din,
yung eye kung natatakpan diba? Ganito. Kapag nilagay mo mapipilitang umayos ang bad eye
yung occluder at hindi nagmove yung uncovered eye, pwedeng
normal to. Kapag may phoria, makikita mo yung REFIXATION Types of Ambylopia
MOVEMENT once you remove the occluder. Meaning pag
tinanggal mo na yung occluder mukhang magfofocus yung Strabismic Ambylopia
tinakpan mong eye para mag-focus • Due to Strabismus
• Management:
Alternate Cover Test o Correct with spectacles
o Patching – occlusion of unaffected eye
o Surgery:
▪ Recession (Weakening)- moving
muscle insertion further back on the
globe
▪ Resection (Strengthening)- shortening
the muscle
o In the case of Esotropia, you weaken the medial
rectus. Kasi pinupull ng MR yung eyeball papunta
sa gitna.
o Botulinum toxin for single muscle weakening
after ocular alignment is restored (via spectacles,
surgery, or botulinum toxin)

TRANS BY: VIRATA AND SANTOS Page 33 of 51


OPHTHA FINALS TRANS

Refractive (anisometric) amblyopia Hering’s Law


• Ambylopia usually occurs in the more hyperopic eye • Yoke’s Muscles
• Due to anisometropia • Corresponding agonist EOMs receive equal innervations
• Management: for eye movement in the same direction
o Treat with glasses to correct refractive error • Example: Left LR & Right MR for a left horizontal gaze
o Patching is required if difference in the visual
acuity persists after 4-8 weeks of using spectacles For both eyes na to. Pag tingin niya sa right, equal innervation
ang nakukuha ng right lateral rectus and left medial rectus to
Deprivation amblyopia (form and occlusion) allow eye movement to the right. At the same time, meron equal
• Due to visual deprivation inhibition sa right medial rectus and left lateral rectus para hindi
• Ex. Congenital Cataract, Periocular hemangioma, Ptosis, naman sila istorbo sa smooth movement ng eye to the right
Retinoblastoma, Occlusion (from patching the good eye)
• Management: Motor Fusion
o Manage or treat the etiology of the occlusion • The process by which the activity of the extraocular
muscles is adjusted to maintain the necessary ocular
• Preferred Eye- retains normal visual acuity alignment for sensory fusion.
• Non-preferred Eye- develops amblyopia (decreased visual • Stimulated by disparity in images received from the two
acuity) eyes, such as results from the object of regard moving
toward or away from the subject
EXTRAOCULAR MUSCLES: MOTOR PHYIOSOLOGY
• The precise action of a muscle depends on the orientation Development of Binocular Movement
of the eye in the orbit & the influence of the orbital
connective tissues • The neuromuscular system of an infant is immature, so that
• Gross extraocular muscle movements: it is not uncommon in the first few months of life for ocular
o Adduction – Directing the eye TOWARD the nose alignment to be unstable.
o Abduction – Directing the eye LATERALLY. • Transient exodeviations are most common and are
o Elevation – Directing the eye UP. associated with immaturity of the accommodation-
o Depression – Directing the eye DOWN. convergence system.
o Intorsion – Directing the TOP of the eye • Gradually improving visual acuity together with maturation
TOWARD the nose. of the ocular motor system allows a more stable ocular
o Extorsion – Directing the SUPERIOR aspect of alignment by age 2 to 3 months.
the eye AWAY from the nose.
• Movement of both eyes together also have two distinct SENSORY CHANGES IN STRABISMUS
types : • If strabismus starts early ( <7 or 8 years old), abnormal
o Version – Movement of both eyes in SAME binocular vision may occur
direction 1. Diplopia
o Vergence – Movement of both eyes but each eye o The same object is seen in 2 different places
moves in an OPPOSITE direction o 1 falls on fovea, 1 falls in peripheral retina
▪ Convergence – directing the left and 2. Visual confusion
right eyes TOWARD the nose o Localization of spatially separate objects to the
▪ Divergence – directing the left and right same location
eyes AWAY from the nose

FIELD OF ACTION
• The position of the eye is determined by the equilibrium
achieved by the pull of all six extraocular muscles. The eyes
are in the primary position of gaze when they are looking
straight ahead with the head and body erect
• 1° position of gaze – eyes look straight ahead with head and
body erect
A. Confusion B. Diplopia
• Field of action of EOM – direction of gaze where the EOM
exerts its greatest contraction force as an agonist
3. Abnormal (anomalous) retinal correspondence
• Synergist Muscle- EOMs within the same eye with the
(ARC).
same field of gaze
o In the presence of manifest strabismus, an
• Yoke muscle- paired EOMs in both eyes that work together extrafoveal retinal locus may become the
to a certain gaze direction preferred point of fixation in the deviating eye,
resulting in abnormal (anomalous) retinal
LAWS IN MOTOR PHYSIOLOGY correspondence (ARC).
Sherrington’s Law o ARC is present only under binocular viewing
• “Share” an eye conditions, in contrast to eccentric fixation (see
• Synergistic and Antagonistic Muscles below). ARC avoids diplopia and visual
• Reciprocal innervations of antagonistic EOMs confusion because the extrafoveal retinal locus of
• Antagonist is inhibited, while agonist is stimulated fixation in the deviating eye is localized straight
ahead during binocular viewing. It also facilitates
Eto yung law for one eye lang. If the patient is looking to the binocular function, possibly resulting in low-grade
right, right lateral rectus ang nagwowork. stereopsis

TRANS BY: VIRATA AND SANTOS Page 34 of 51


OPHTHA FINALS TRANS

4. Suppression • No restriction in range of eye movements


o In binocular vision, • Monocular, alternating, or intermittent
▪ Dominant eye: seen
▪ Strabismic/amblyopic eye: not perceived NEURO-OPHTHALMOLOGY
“suppressed” VISUAL PATHWAY
o In the absence of strabismus, a blurred image in 1
eye may also lead to suppression
o The lack of simultaneous perception in
the central retina prevents fire
stereopsis (from the peripheral retina)
may still be present

5. Amblyopia
• Decreased visual acuity without any organic
disease in 1 eye

STRABISMUS TYPES
Paralytic Strabismus
• Incomitant strabismus
o The deviation varies in different positions of gaze
• Mostly in adults, acquired
• Due to reduction or restriction in range of eye movements
Visual Field Defect Lesion
1 Total blindness (left eye) Optic nerve
(Complete; left)
2 Bitemporal hemianopsia Optic chiasm
3 Right homonymous Optic Tract
hemianopsia
4 Right homonymous superior Temporal Lobe
quadrantanopia “pie in the sky”
5 Right homonymous inferior Parietal Lobe
quadrantanopia “pie in the
floor”
6 Right homonymous Left Occipital lobe
hemianopia with macular
spring
On primary gaze, the patient is right esotropic. No deviation on PEARLS
left gaze. But when he looks to the right, hindi maka abduct yung • Hemianopias should have intact visual acuity in the
right eye. Anong muscle ang affected? Right Lateral Rectus spared visual field
• Lesions that are: Must know!
o Anterior to the Optic chiasm – UNILATERAL
o At the chiasm – BITEMPORAL HEMIANOPSIA
usually pituitary tumor
o Posterior to the chiasm – CONTRALATERAL
HEMIANOPIA/QUADRANTONOPIA
• More congruous lesions(more similar in size, shape and
Eto naman right exotropic on primary gaze. No deviation on location) = more posterior lesion
right gaze. On left gaze, hindi naman maka- adduct yung right • Optic tract lesions vs. Occipital lobe lesions
eye. Ano ang affected? Right medial rectus o Optic tract = incongruous homonymous
defects
Non-Paralytic Strabismus o Occipital lobe = identical defects in each field
• Comitant strabismus- The deviation is equal in all
directions of gaze
• Usually begins in infancy

TRANS BY: VIRATA AND SANTOS Page 35 of 51


OPHTHA FINALS TRANS

Left-Pale nerve Right-Optic Atrophy

Optic Nerve Hemorrhage


• If located in the nerve fiber layer (NFL) of the retina, they
are splinter or flame-shaped hemorrhages
• If located deeper, disc hemorrhages may appear round and
OPTIC NERVE
blotchy
• Axons can be dysfunctional long before the optic nerve
• Two theories: Mechanical (shearing forces) and vascular
becomes atrophic
(ischemia)
• Optic nerve in the canal is most susceptible to damage
Mechanical
Optic Disc Edema
• disc hemorrhages result from mechanical shearing at the
• Predominantly in diseases directly affecting the anterior
lamina cribrosa or because of damage to the capillary
part of the optic nerve (such as in increased intracranial network at the border of retinal NFL defect enlargement
pressure and compression of the intraorbital optic nerve)
Vascular
• Papilledema – optic disc edema caused by an increase in
• ischemic microinfarction in the optic nerve head or
intracranial pressure
perturbation of the blood-retinal barrier

• Papilledema – usually normal vision vs. Optic neuritis – 1st picture: Inflammatory conditions or possible central vein
often associated with blurred vision occlusion
• In very severe papilledema – florid hemorrhage and 2nd picture (red arrow): Splinter hemorrhage – rule out
disorganized posterior pole glaucoma
3rd Picture (red arrow): Splinter hemorrhage
Optic Atrophy
OPTIC CHIASM
• Non-specific response to optic nerve damage from any
• Chiasmal lesions cause Bitemporal Hemianopsia.
cause including retinal diseases (such as retinitis
• At an EARLY stage, field defects are incomplete and
pigmentosa and CRAO)
asymmetric.
• May be a sign of prior disc edema
• At a LATER stage, the temporal field defects become
• May be a late sign of compressive optic neuropathy
complete.
• Presentation is always PALLOR disk
OPTIC CHIASM PATHOLOGIES
• Pituitary Adenoma – common in ADULTS
• Craniopharyngioma – common in CHILDREN

THE LATERAL GENICULATE BODY


THE RETROCHIASMATIC VISUAL PATHWAY
• Cerebrovascular disease and tumors are the MOST
COMMON lesions of the retrochiasmatic visual pathways.
• Due to their multiple vascular supply, the optic tracts and
LGN are rarely affected by vascular lesions.
• Retrochiasmatic field defects are homonymous.
• In patients above 50 years old, vascular lesions are the
most common (80%) cause of occipital lobe conditions.

TRANS BY: VIRATA AND SANTOS Page 36 of 51


OPHTHA FINALS TRANS

• The pattern of field defect depends on the affected area of RELATIVE AFFERENT PUPILLARY DEFECT
the occipital lobe. • Also known as Marcus Gunn Pupil
• Macular sparing occurs due to the dual blood supply of • Sensitive and specific sign to detect optic nerve pathology
the OCCIPITAL LOBE: or large retinal pathology
o Posterior cerebral artery: main blood supply • “Relative” – in relation to the contralateral eye
o Branches of the middle cerebral artery(MCA): • “Afferent” – refers to the afferent arm of the pupillary light
supplies the occipital lobe tip (responsible for the pathway(optic nerve)
central macular vision)

Macular sparring: Bakit nasspare ang macula when you


have occipital lobe lesion?
The main blood supply of the eye is your post cerebral artery
(PCA) but your central vision receives branches from the MCA.
So if the px suffers from stroke involving just the PCA, you still
have perfusion from the MCA. So yung affected lang are the
areas subserved by your PCA. Kaya macula is sparred.
• LEFT HOMONYMOUS HEMIANOPSIA WITH MACULAR
SPARING

• Detected using swinging flashlight test


• When shine a light to right eye, expect the left eye to
constrict.
AFFERENT AND EFFERENT LESIONS

Where is the lesion in the visual pathway? Occipital lobe,


most probably involving the PCA. Right or left? Right. Kasi
contralateral. What if the MCA is involved? Macular Scotoma
naman. You see, it can vary depending on the location
involved.
• HOMONYMOUS MACULAR SCOTOMA

PUPILLARY NEAR REFLEX


THE PUPILS • Triad:
• 20-40% of normal patients have physiologic Anisocoria o Convergence
(~0.5mm). o Accommodation
• Anisocoria- the pupils are asymmetric, one pupil is larger o Miosis
than the other.
THE PUPILLARY LIGHT REFLEX ANISOCORIA
• Afferent arm Cranial Nerve II (Optic Nerve) • Urgent condition! Inequality in the size of the pupil
• Efferent arm Cranial Nerve III (Oculomotor Nerve) • Something that the patient might be born with (physiologic
• Visual stimuli through a branch going to the midbrain will anisocoria; usually >1-2 mm)
cause pupillary constriction
PUPILLARY LIGHT NEAR DISSOCIATION
ANISOCORIA CONDITION • Miosis at near > miosis to light
Anisocoria in Dark > Horner’s Syndrome • Occurs in lesions that affect the ciliary ganglion or the
Anisocoria in Bright midbrain, in which the light reflex pathway is relatively
Anisocoria in Bright > Adie’s Tonic Pupil dorsal compared to the near response pathway.
Anisocoria in Dark
Anisocoria in Dark= Anisocoria Physiologic Anisocoria ARGYLL ROBERTSON PUPILS
in Bright= Anisocoria in Near • PROSTITUTE’S PUPIL
Response • Accommodates, but does NOT react
Anisocoria in Dark= Anisocoria Light Near Accomodation
• Small (<3mm) pupils, irregular, eccentric
in Bright < Anisocoria in Near
Response o Difficult to dilate due to iris atrophy
Anisocoria +- CN3 Palsy Aneursym • Highly suggest tertiary (CNS) syphilis

TRANS BY: VIRATA AND SANTOS Page 37 of 51


OPHTHA FINALS TRANS

ADIES’S TONIC PUPIL Which one has Adie’s? The right eye – because it’s dilated kasi
• Due to damage of the ciliary ganglion (in the orbit) or nga poor constriction
short ciliary nerves
• Etiology: autonomic neuropathy, after retinal laser
photocoagulation
• In 50% of cases, the contralateral eye may be involved in
the next 10 years.
• Denervation hypersensitivity to 0.125% pilocarpine
(pupils constrict to weak pilocarpine)
• Early stage: pupil is dilated, accommodation is impaired
• Later stage: accommodation recovers, incomplete
reinnervation of the iris
o Segmental iris constriction (more constricted than
the normal contralateral eye)
o Pupil LND
• ADIE-HOLMES: Adie’s tonic pupil + loss of DTR
• How do we test for Adie’s? We use pilocarpine (0.125%) –
pupils constrict. HORNER’S SYNDROME
• Due to damage to the oculosympathetic pathway
• Triad:
o Ptosis
o Miosis
o Anhydrosis

• 10% cocaine to CONFIRM presence


• 1% hydroxyamphetamine to LOCALIZE
• Adie’s – difficulty in constriction, so pupils are expected to
bedilated
• Horner’s – pupils are expected to be constricted. If
Horner’s is confirmed, we have to localize (pre-ganglionic
–2nd order, post-ganglionic – 3rd order). We use 1%
hydroxyamphetamine. We instill this drug, if pupils dilated
= pre-ganglionic, if pupils did not change in size = post-
ganglionic.

The OCULOSYMPATHETIC PATHWAY


• 1st order (central): Posterior hypothalamus to C8-T2
• 2nd order (pre-ganglionic): C8-T2 to superior cervical
ganglion
• 3rd order (post-ganglionic): Superior cervical ganglion to
the carotid plexus and CNV1 (trigeminal), which enters the
orbit
• A lesion at any site along the pathway can produce Horner’s
syndrome

TRANS BY: VIRATA AND SANTOS Page 38 of 51


OPHTHA FINALS TRANS

CRANIAL NERVES PALSIES CASES NA LAST NA PUSH MO NA GO GO GO!


CRANIAL NERVE III PALSY (OCULOMOTOR) CASE 1: JUNCTIONAL SCOTOMA
A 56-year-old woman reports progressive decreased
• Review: “LR6-SO4-AO3”
vision in her left eye for the past 3 months. Her past ocular
• Complete or incomplete history is unremarkable. On ocular examination, her visual
o Complete: SR, MR, IR, IO, levator palpebrae acuity is 20/20 OD and 20/100 OS. Extraocular motility is intact,
superioris/ there is (+) ptosis confrontation visual fields (VF) appear full, and there is a
o Incomplete: SR, MR, IR, IO / (-) ptosis positive RAPD in the left eye.
• Pupil-involving or pupil-sparing
o (+) dilated pupil: Request for a cranial MRI to rule
out cerebral aneurysm
o Normal pupil: most likely ischemic only

1. What do you notice about the optic nerve?


• The optic disc has pallor horizontally.

2. What additional testing would need to be ordered in


order to further evaluate the suspicious findings on ocular
examination?
• Visual Field Testing
CRANIAL NERVE IV PALSY (TROCHLEAR) o Tangent screen
• 40% trauma – most common cause
• Expect head tilt and hyperopia. Usually guy with hyperopia,
nakataas yung right eye, yung head tilt nya is on the
opposite side, then you have right CN IV palsy.

o Humphrey field analyser


o Goldmann

CRANIAL NERVE VI PALSY (ABDUSCENS)


• Lateral rectus is not functioning, so patient has Esotropia
because there is paralysis of LR. When you ask this patient • Color Vision Testing
o Color Vision Testing showed: color vision is normal
to look to the right, hindi nya magagalaw. An esotropic
OD and reduced OS.
patient with no LR palsy can still look to the right.
• You have to rule out intracranial tumors, lalo nay ung sa
base ng brain kasi usually yung CN VI is seen at the base
of the brain.

• Visual Field Analysis shows:

TRANS BY: VIRATA AND SANTOS Page 39 of 51


OPHTHA FINALS TRANS

3. Does the Visual Field test localize the pathology and, if • Anterior chamber reaction
so, where is the lesion located? • Vitreous
• The Visual Field defect is a junctional scotoma due to a • Retina
lesion of the optic nerve near the chiasm, which involves • Optic nerve
the knee of von Willebrand (inferonasal retinal fibers that
cross in the chiasm and then travel anteriorly approximately 3. What is the diagnosis at this stage?
4mm into the opposite (contralateral) optic nerve before Retrobulbar Optic Neuritis
running posteriorly to the brain) causing central visual loss
in the ipsilateral eye and a superotemporal field defect in • An inflammation affecting the optic nerve behind the eyeball
the contralateral eye. without ophthalmoscopic manifestations in the fundus.
• It is characterized by:
o Sudden decrease in vision that rapidly progresses to
4. What would be the differential diagnosis at this stage? visual loss
Chiasmal syndrome o The acute phase of this disease is usually unilateral
• Which is most commonly a mass lesion but possibly due to whereas, its chronic condition (toxic amblyopia) is
a hemorrhage. The differential diagnosis includes tumors, generally bilateral.
pituitary apoplexy, aneurysm, trauma, sarcoidosis, and o Duke-Elder states, “In the majority of cases the
chiasmal neuritis. fundus appears normal and the condition may be
5. What additional work up would be necessary to arrive at defined as a disease in which neither the examiner
the final diagnosis? nor the patient sees anything.”
• Signs and Symptoms: usually worsen for 2 weeks and then
• Neuroimaging
stabilize.
• Also consider endocrine evaluation (check hormone levels). o Sudden decrease of vision
o Eye pain (probably due to pressure)
CASE 2: RETROBULBAR OPTIC NEURITIS o Blurred/dimmed vision
A 22-year-old woman reports decreased vision and eye o Pain with eye movement
pain in the left eye that started 2 days ago and has gotten
worse. She says everything looks blurry and appears dimmer
on the left eye compared to other eye. The optic disc on the left 4. What additional testing would you perform and why?
is shown below: • Check color vision
• Visual fields
• Perform a head/orbital MRI to look for periventricular white
matter demyelination lesions or plaques (the best predictor
of future development of multiple sclerosis)
• Neurologic examination

5. What are the recommendations for treatment?


• According to the Optic Neuritis Treatment Trial (ONTT), if
the MRI is positive, then consider systemic steroids
(methylprednisolone 250 mg IV q6h for 3 days, followed by
She denies any trauma or previous similar episodes. Her prednisolone 1 mg/kg/day for 11 days and rapid taper 20
past medical history is negative and she takes only birth control mg/day on day 12 and 10 mg/day on days 13-15).
pills. The eye hurts especially when she looks to the right and • Visual recovery occurred 2 weeks faster with this regimen
left. There is no redness, photophobia, or diplopia. She has no than with other treatments; however, there was no
other associated symptoms. difference in the final visual acuity.
Her ocular exam is normal except for 20/200 vision on the • Furthermore, there was a decreased incidence of MS over
left eye and a positive RAPD on the left eye. the next 2 years but there was no difference after 3 years.
• Oral steroids should not be used alone, because this led to
1. What would you ask this patient? an increased risk of recurrent optic neuritis.
• What type of eye pain (foreign body sensation (FBS), sharp, • Additional treatment recommendations showed that
dull, ache, tender to touch, pain with eye movement, patients who received weekly intramuscular inferferon β-
headache, periorbital pain)? 1a (Avonex) after steroid therapy for the first episode of
• Any double vision? optic neuritis associated with two lesions on MRI greater
• Any trauma? than 3 mm had a reduction in onset of clinical MS over 3
• Any previous episodes? years and improvement or less worsening of MRI lesions.
• Has the eye been red?
• Any sensitivity to light? 7. What is the prognosis?
• Any other neurologic symptoms? ● Most patients recover vision over the following months, but
• Does she have any medical conditions? the final acuity depends upon the severity of the visual
• Is she taking medication? loss. The majority of patients recover 20/20 vision;
however, permanent subtle color vision and contrast
sensitivity deficits are common. Uhthoff’s symptom
2. What would you specifically look for on exam?
(blurred vision with increased body temperature or
• Visual Acuity exercise) may occur. Approximately 30% of patients with
• Pupillary response with attention to the presence of a RAPD isolated optic neuritis will experience another attack in
• Extraocular motility either eye, and up to 50% of patients will develop MS over
• Cornea 5-10 years.

TRANS BY: VIRATA AND SANTOS Page 40 of 51


OPHTHA FINALS TRANS

CASE 3: PRIMARY CONGENITAL GLAUCOM • Surgical options include goniotomy (children <1.5 years
A mother brings in her 5-month-old boy because his eyes old with clear cornea) and trabeculectomy (cloudy cornea,
have been tearing for a couple of months. On further children >1.5 years old, or two failed goniotomies)
questioning, she reports no discharge or redness, but he • If these fails, then trabeculectomy with mitomycin C,
squints and turns away from bright lights. He has no glaucoma drainage implant, and cycloablation are
significant past ocular or medical history. options.
• Correction of any refractive error and treatment of
amblyopia must also be performed.

CASE 4: FOLLICULAR CONJUNCTIVITIS


A 24-year-old woman reports red, itchy eyes for the past
month. Her primary physician treated her with gentamicin eye
drops for 2 weeks with no improvement. She was then given
Blephamide for 1 week, which helped, but her symptoms
recurred when she stopped the drops.
1. What ocular findings would you look for at this stage? She has not used any eye drops for 1 week. She denies
• Increased lacrimal lake any change in vision or any mucus discharge from her eyes.
• Discharge from lacrimal puncta Gross photo is shown below:
• Corneal staining
• Foreign body on the ocular surface or tarsal conjunctiva
• Anterior chamber cell and flare
• Buphthalmos
• Myopia
• Enlarged corneas
• Increased IOP
• Optic disc cupping
1. What is the diagnosis?
Gross slit lamp exam findings are shown below: Follicular conjunctivitis
• This is seen in a variety of conditions, including
inflammation caused by pathogens such as viruses,
atypical bacteria, and toxins, including topical
medications (eg. Glaucoma medications, especially
brimonidine, or OTC ophthalmic decongestants)
• Follicles are small, dome-shaped nodules without a
prominent central vessel
o Appears paler on its surface and redder at its base.
o Most prominent in the inferior palpebral and
forniceal conjunctiva.
2. What finding is depicted in the slit lamp photo and what • Histologically, a lymphoid follicle is situated in the
is the diagnosis? subepithelial region and consists of a germinal center
Primary Congenital Glaucoma containing immature, proliferating lymphocytes, and
surrounding corona containing mature lymphocytes and
• Slit lamp photo
plasma cells.
o Haab striae
• Gross findings:
▪ Curvilinear breaks in Descemet's membrane,
o Follicles appear as gray-white, round to oval elevations
resulting acutely from stretching of the cornea
measuring between 0.5 to 1.5 mm in diameter
in primary congenital glaucoma.
o Smaller follicles may produce a slightly irregular and
velvety appearance
3. What would you tell the mother about the diagnosis? o Follicles can be seen in the inferior and superior tarsal
• Congenital glaucoma occurs in the first few months of life, conjunctiva, and less often, on bulbar or limbal
is usually bilateral, and can be primary (developmental conjunctiva.
abnormality of the angle structures, most commonly due to
a mutation in the CYP1B1 gene), secondary, or associated 2. What are the possible etiologies causing this condition?
with ocular or systemic syndromes.
• Virus (adenovirus, HSV)
• An examination under anesthesia may be required to check
• Chlamydia
tonometry, corneal diameter, gonioscopy, and
ophthalmoscopy. • Molluscum
• Drug reaction
4. What is the recommended treatment for this disease?
3. What are the etiologies of membranous conjunctivitis?
• Treatment is surgical
• Topical medications (beta- blockers or carbonic • A true membrane is caused by Streptococcus,
anhydrase inhibitors) are used to control IOP until surgery Gonococcus, Corynebacterium diphtheriae, Steven-
can be performed Johnson syndrome, and chemical burns.
• Brimonidineis contraindicated because it can be
associated with death

TRANS BY: VIRATA AND SANTOS Page 41 of 51


OPHTHA FINALS TRANS

4. What other questions would you ask this patient at this 2. How would you manage this patient?
point? • Perform corneal cultures and smears of the ulcer and start
• Has she had a recent upper respiratory infection of fever? empiric antibiotic treatment with topical fortified antibiotics
• Has she been around anyone with an eye infection? (cefazolin and tobramycin) alternating every hour initially,
• Does she wear contact lenses? and a cycloplegic drop.
• Has she ever had a sexually transmitted disease? • Ask the patient about contact lens wear and if he does wear
contacts the lens and case should be cultured as well.
5. What other findings would you look for on exam? • This vision-threatening corneal infection initially required
daily follow-up monitoring the vision, IOP, size, depth, and
• Preauricular lymphadenopathy density of the corneal infiltrate, and presence and size of
• Eyelid lesions any overlying epithelial defect.
• Pseudomembrane on the inferior tarsal conjunctiva • Therapy should be adjusted based on the culture and
• Subconjunctival hemorrhage sensitivity results.
• Subepithelial infiltrates in the cornea • Confocal microscopy if available is useful for identifying
fungi and Acanthamoeba.
Additional information: She recently had a flare-up of sinusitis
but has not had a cold or been around known contacts with an
3. What are the indications for a corneal biopsy?
eye infection. She does not wear contact lenses and does not
have any known allergies. She denies any history of having • A biopsy should be considered for progressive disease, a
STD. There are no eyelid lesions, no subepithelial corneal culture negative ulcer, or a deep abscess.
infiltrates, and no preauricular lymphadenopathy.
Additional information: KOH prep is positive and a Gram’s
stain of scraping from the ulcer shows the following organism:
6. How would you work up this patient?
• Obtain a conjunctival culture and scraping
• Initiate empirical antibiotics

Additional information: The conjunctival scraping from the eye


shows the following picture:

4. What is the diagnosis?


Fungal Keratitis
• The scarping shows branching fungal hyphae of Fusarium.

5. What are the characteristic findings of this type of


7. What is the diagnosis? keratitis?
Chlamydial conjunctivitis • Fungal ulcers usually have feathery edges, endothelial
• The scrapings (photo above) shows basophilic plaque, satellite infiltrates, and may penetrate Descemet’s
cytoplasmic inclusion bodies in epithelial cells. membrane.

8. What is the recommended treatment? 6. What is the recommended treatment?


• Systemic and topical antibiotics with tetracyclines or • Topical antifungal (Natamycin, Amphotericin B, miconazole
erythromycins (azithromycin), and also treat all sexual or voriconazole)
partners. • Cyclopegic drops (Atropine sulfate 1%)
CASE 5: MICROBIAL KERATITIS • Topical steroids are contraindicated.
An 18-year-old man complains of a red and painful right • For severe infection, add systemic antifungal (ketoconazole
eye with blurry vision. It has gotten worse over the past 4 days or amphotericin B.
since he was gardening and thinks he got scratched by a CASE 6: GIANT PAPILLARY CONJUNCTIVITS
plant branch. A 26-year-old woman complains of red, irritated eyes for
3 weeks. She wears disposable contact lenses but cannot
tolerate them for more than an hour or two. Yesterday she
noticed some mucous discharge in the morning.

1. What is the most likely diagnosis?


Microbial keratitis
• Probably fungal or bacterial.

TRANS BY: VIRATA AND SANTOS Page 42 of 51


OPHTHA FINALS TRANS

Her primary doctor gave her an antibiotic ointment, which


she puts in her eyes twice a day, and she also uses artificial
tears once or twice a day. Ocular Exam shows scant white
mucus strands in the inferior fornix OD and minimal
conjunctival injection OU.

Additional information: Ocular Examination of the eyelids and


tear film are normal. There are no conjunctival follicles on the
bulbar or inferior palpebral surface, and there is no corneal 1. What condition does he have and what other ocular
staining or infiltrate. problems are associated with it?
• Pseudoexfoliation syndrome (PXS), which is associated
2. Approach to Diagnosis? with:
• Giant papillae on the superior tarsal conjunctiva by everting o Angle closure
the upper eyelids. o Ectopia lentis
o Ocular hypertension
Additional information: This maneuver on ocular exam reveals o Glaucoma (PXG)
the following on the slit lamp:
2. What other details from the history would be helpful?
• What is his past ocular and medical history?
• Does he have any other ocular problems or has been
treated for any eye conditions?
• What medication does he take? Specifically, does he take
medicine for his prostate or blood pressure?

3. What would you pay particular attention to an ocular


examination?
• Gonioscopy
3. What is the diagnosis? • Diameter of dilated pupil
Giant Papillary Conjunctivitis • Iridodonesis
• Phacodonesis
• It is a chronic inflammatory process that leads to
• Corneal pachymetry
production of giant papillae at the tarsal conjunctiva.
• IOP
• It is usually caused by an allergy to contact lenses or to
• Optic nerve appearance
the chemicals that are used to clean them, therefore, soft
contact lens wearers have a great chance of getting it.
• The proteinaceous deposits on the contact lenses serve 4. What specific risks does this patient have with respect
as the antigen that causes the pseudo-allergic reaction. to treatment for cataracts?
• The tarsal conjunctiva becomes irritated that it becomes • There is an increased risk of complications due to both the
red, swollen and rough which may soon lead to the pseudoexfoliation (weal zonules) and Flomax (IFIS,
formation of papillae. intraoperative floppy iris syndrome).
• Other symptoms include feeling like something is stuck in • He should be warned that theses conditions increase the
the eye, painful and itchy eyes, swollen or droopy eyelids, chance of complications such as posterior capsular rupture,
and excess mucous production that may cause blurring of vitreous loss, retained lens fragments, alternate lens
vision. placement, late lens dislocation, iris damage, and
misshapen pupil.
4. How would you treat her? • He should also be informed that PXS increases his risk for
glaucoma (PXG) in the future, even if the lens is removed.
• Treatment is to discontinue contact lens wear.
• Use a combination of mast cell stabilizer/antihistamine drop
• Depending on the severity, also add a topical steroid. If a 5. What is IFIS?
steroid is used, the IOP must be monitored. • IFIS is a condition characterized by a variable degree of
poor pupillary dilatation, and floppy/atonic iris that
5. Beside contact lenses, what are other causes of this billows, prolapses, and becomes miotic during surgery.
disorder?
• GPC is mainly associated with contact lens wear, but other 6. How is IFIS managed?
causes include a foreign body, exposed suture, or • Mild forms may respond to preoperative topical atropine
prosthesis. and intraocular preservative-free epinephrine. Most
severe forms require iris stabilization with one or more of
CASE 7: PSEUDOEXFOLIATION SYNDROME the following:
A 78-year-old man has a visually significant cataract and o Healon 5
desires surgery. On exam, he dilates poorly and his lens o Pupil expanders (maylugin ring, graether ring,
appearance is seen in the photo. Iiring)
o Iris hooks
o Low-flow fluidics during phacoemulsification

TRANS BY: VIRATA AND SANTOS Page 43 of 51


OPHTHA FINALS TRANS

CASE 8: SYPHILIS 5. What other ocular findings occur in this disease?


A 52-year-old asymptomatic woman comes in for a
complete eye exam. An anterior segment exam shows the Ocular Description Image
finding in the photo. A posterior segment exam shows mild findings
attenuation of the retinal vasculature and diffuse
pigmentary changes in both eyes. Interstitial Nonsuppurative
Keratitis inflammation
characterized by
cellular infiltration of
the corneal stroma
that may lead to
scarring. This could
be to infectious or
immunologic cause.

Ectopia lentis Misplacement of


malposition of the
1. What questions would you ask the patient at this stage? crystalline lens of
● Ask History of disease, infection or trauma the eyes due to
● Ask if she had an episode of a red eye with blurry vision or disruption or
photophobia dysfunction of the
● Ask if there is any past medical history of Arthritis, zonular fibers of the
lens secondary to
Autoimmune disorders or Infectious disease
trauma or heritable
● Ask if she ever had a sexually transmitted disease
condition.

Additional information: She reports a few episodes of Argyll- A small pupil that
“conjunctivitis” for which she was given antibiotic and steroid Robertson constricts poorly to
drops. Cultures or additional workup were not performed. Her pupil direct light but
past medical history is positive for Chlamydia at age 28 and briskly reactive
hypercholesterolemia. within reading
distance is viewed
(“light-near
2. What would you do next as part of the General dissociation”)
Screening for this condition?
• Complete Blood Count Journal:
• Rheumatoid Factor Localization of the
AR will depend on
• Antinuclear Antibody
careful examination
• Angiotensin converting enzyme
of patients with
• VDRL or RPR (non-treponemal test) bilateral
• FTA-ABS or MHA-TP (treponemal test) neuropathic tonic
• PPD and controls pupils for syphilis
• CXR and favor checking
the blood of the
Additional information: the tests are negative except for a patients (Thompson
positive VDRL and FTA-ABS. & Kardon, 2006)

Chorioretinitis Yellowish, ill-


3. What treatment would you prescribe and why? defined, placoid
● Lumbar puncture lesion in the
○ To rule out neurosyphilis posterior pole or
mid-periphery of the
● Systemic Penicillin
fundus.
○ Indicated for the patient and her sexual partners (may
use Tetracycline if allergic to Penicillin) Usually occurs in
● VDRL/RPR the secondary stage
○ To monitor treatment efficacy of syphilis

4. What is the causative organism? Fundoscopy of right


eye showing
Treponema pallidum yellowish white
• A spirochete which causes a complex, systemic, infectious retinal lesions
disease.
Optic Atrophy Refers to the death
of the retinal
ganglion cell axons
that comprise the
optic nerve. Pale
optic nerve on
fundoscopy

TRANS BY: VIRATA AND SANTOS Page 44 of 51


OPHTHA FINALS TRANS

4.) How is this condition distinguished from the


mesodermal dysgenesis syndromes?
6. What are the systemic signs of the congenital form of • Iridocorneal endothelial syndrome is a unilateral,
this illness? nonhereditary, progressive abnormality of the corneal
• Saber shins, saddle nose, peg teeth, internal organ endothelium that is not associated with any systemic
inflammation abnormalities.
• It most commonly affects middle-aged women.
• Mesodermal dysgenesis is bilateral, congenital, and
hereditary.

5.) What are the mesodermal dysgenesis syndromes and


findings of each?
a. Axenfeld’s anomaly
• Posterior embryotoxon (anteriorly displaced Schwalbe's
line) with iris processes to the scleral spur. Glaucoma
develops in 50%.
Figure A. Saber shin Figure B. Saddle nose Figure C. Peg teeth
b. Alagille’s syndrome (ALGS)
CASE 9: IRIDIOCORNEAL ENDOTHELIAL SYNDROME • Axenfeld's plus pigmentary retinopathy, corectopia,
A 33-year-old woman complains of double vision on her esotropia, and systemic abnormalities (absent deep tendon
left eye. reflexes, abnormal facies, pulmonic valvular stenosis,
Gross photo of her left eye is shown below: peripheral arterial stenosis, biliary hypoplasia, and skeletal
abnormalities).

c. Reiger’s anomaly
• Axenfeld’s plus iris hypoplasia with holes. Glaucoma
develops in 50%.

d. Reiger’s Syndrome
• Reiger’s anomaly plus mental retardation and systemic
abnormalities (dental, craniofacial, genitourinary, and
skeletal).
1.) What condition do you suspect she has? e. Peter’s Anomaly
• ICE syndrome (iridocorneal endothelial syndrome), • Central corneal leukoma (opacity due to defect in
specifically essential iris atrophy. Descemet's membrane with absence of endothelium) with
iris adhesions, may have cataract and develop glaucoma
2.) What are the characteristic findings of each disorder in (50%), and is associated with cardiac, craniofacial, and
this syndrome? skeletal abnormalities. It is usually sporadic and bilateral
IRIS NEVUS (COGAN-REESE) SYNDROME (80%).
CASE 10: SUNSET SYNDROME
• Iris nevus (Cogan-Reese) syndrome: flattening and
A 79-year-old woman was previously diagnosed with
effacement of the iris stroma, pigmented iris nodules
pseudoexfoliation syndrome status post uncomplicated cataract
(pseudonevi) composed of normal iris cells that are
surgery 6 years ago. Recently, she notices increasing blurry
bunched up from the overlying membrane and ectropion
vision after bumping her forehead on a towel rack in the
uvea.
bathroom 1 week ago. Findings shown in the photo:
CHANDLER SYNDROME
• Corneal edema often with normal IOP, and mild or no iris
changes (minimal corectopia, iris atrophy, peripheral
anterior synechiae).
PROGRESSIVE IRIS ATROPHY
• Proliferating endothelium produces broad PAS, corectopia
ectropion uveae, and iris holes (stretch holes [area away
from maximal pull of endothelial membrane in stretched so
thin that holes develop] and melting holes (holes in areas
without iris thinning due to iris ischemia]).
Question 1. What is the term for the finding demonstrated
in the photo?
3.) What is the pathophysiology of this disease?
• Sunset syndrome
• Abnormal corneal endothelium grows across the angle and
iris, obstructs the trabecular meshwork, distorts the iris, and
Question 2. What are the etiologies?
contracts around the iris stroma to form nodules.
A. Bag IOL (intraocular lens)
• Zonulysis from trauma/ Zonular dehiscence or disruption is
the most frequent cause of the sunset syndrome.
• Conditions that can cause ectopia lentis.

TRANS BY: VIRATA AND SANTOS Page 45 of 51


OPHTHA FINALS TRANS

B. Sulcus IOL
• Insufficient capsular support Question 2. What are the notable associations with the
• Inappropriate IOL (length too short for sulcus) history?
• Capsular contraction w/ asymmetric haptic placement (1 in • Anatomic features that predispose to angle closure are
and 1 out of bag) small anterior segment (hyperopia, nanophthalmos,
microcornea, microphthalmos), anterior iris insertion
(Eskimos, Asians, and African Americans), and anterior
Question 3. How would be the optimal treatment for this chamber (large lens, plateau iris configuration, loose or
condition? subluxed lens, pseudoexfoliation syndrome.
IOL repositioning Question 3. What exam findings would you expect to see?
● The hepatics (or capsular tension ring if present) can be • Visual Acuity Testing
sewn to the iris or sclera using a variety of techniques. o Decreased vision
• Pupillary Light Reflex
IOL exchange
o Mid-dilated poorly reactive pupil with possible RAPD
● Depends on the adequacy of capsular support and the (Relative Afferent Pupillary Defect)
status of anterior chamber • Gross Eye Exam
● Lens options: o Conjunctival injections
○ Sulcus IOL w/ or w/o suture fixation o Iris atrophy
○ Iris-fixated IOL • Fundoscopy
○ Anterior chamber IOL o Optic nerve swelling
o Hyperemia
Question 4. After IOL repositioning, the patient • Slit Lamp Examination
subsequently develops a visually significant posterior o Corneal epithelial edema
capsular opacification. What would be the treatment for o Closed angle
this, and what are the possible complications? o Iris bombe
o Shallow anterior chamber
Treatment:
o Mild anterior chamber cells and flare
● An option for PCO treatment is called YAG laser posterior o Slit Lamp Biomicroscopy - Glaukomflecken
capsulotomy which involves dilating the eyes before the • Tonometry
procedure and with the use of laser, removes the hazy o Elevated intraocular pressure
posterior capsule that was brought about by the epithelial • Gonioscopy
cells that regrown after the IOL repositioning. No incision o Peripheral anterior synechiae
is needed but may require further therapy of anti-
Question 4. How do you distinguish between appositional
inflammatory eye drops to reduce further complication and
and synechial angle closure?
haziness.
• Indentation gonioscopy with a Zeiss-style 4-mirror lens.
Indenting the cornea forces aqueous fluid peripherally
Possible complications: toward the angle. The angle opens if closure is
● Increased IOP appositional and remains closed if closure is synechial.
● Iritis
● IOL optic damage Question 5. What provocative tests can be used to
diagnose angle closure?
● IOL dislocation
● Posterior vitreous detachment • To diagnose angle closure, provocative tests include the
prone test, dark-room test, prone dark-room test, and
● Cystoid macular edema
pharmacologic pupillary dilation.
● Corneal or retinal burn • Testing is positive if the IOP rises >8 mmHg.
● Retinal tear & detachment
CASE 11: ACUTE ANGLE CLOSURE GLAUCOMA Question 6. How would you treat this patient?
A 51-year-old woman woke up with decreased vision • This case requires an immediate treatment because its an
and eye pain in the right eye. It is hard for her to open her ophthalmic emergency, the treatment should aim to lower
eyes for the exam and she says she feels nauseous. She is the IOP. The treatment can be the following:
allergic to penicillin and sulfa. The appearance of the • Therapeutic Management:
anterior segment is shown in the photo. o Topical hypotensive drops (for example: beta
blocker, pilocarpine and alpha- adrenergic agonist)
▪ Reduce the IOP by blocking the sympathetic
nerve endings in the ciliary epithelium which
leads to a decrease in aqueous humor
production
❖ Not effective if IOP is >40 mmHG when due
to sphincter ischemia because it will cause
the lens-iris diaphragm to move forward and
will worsen the pupillary block
o Carbonic Anhydrase Inhibitor is contraindicated
to the patient because of her allergy to sulfa
Question 1. What is the diagnosis? o Oral agents like hyperosmotic agent (isosorbide,
● Acute Angle Closure Glaucoma (AACG) glycerin and IV Mannitol)

TRANS BY: VIRATA AND SANTOS Page 46 of 51


OPHTHA FINALS TRANS

▪ Lowers intraocular pressure by reducing the cortical/subcapsular cataract, and optic cupping in the right
vitreous volume eye.
▪ Contraindicated in diabetics (glycerin)
▪ watch for cardiovascular adverse effects (IV
Mannitol)
o Topical steroid
▪ anti- inflammatory
o Topical glycerin
▪ for clearing corneal edema
• Surgical Management:
o Laser Peripheral Iridotomy
▪ definitive treatment
▪ only be performed when the cornea is clear
enough to have an adequate view. Question 1. What additional history would be helpful?
o Surgical Iridectomy • What is his past medical and ocular history?
▪ necessary if laser peripheral iridotomy cannot • Is there a history of trauma, surgery, or steroid use?
be performed • Does he take any medication?
• Does he have a family history of eye disease?
Additional information: when she returns 1 week after the
laser iridotomy for a pressure check, her IOP is 30 mmHg, Question 2. How would you work up this patient?
the iridotomy is patent, and gonioscopy reveals a narrow • Slit-lamp exam with attention to signs of anterior segment
angle. injury (corneal scars, iris and angle tea phacodonesis,
cataract)
Ultrasound biomicroscopy shows: • check IOP
• corneal pachymetry
• gonioscopy
• visual fields
• optic nerve head photos/imaging.
Additional information:
The patient says he was hit with a tennis ball in the right eye
as a child and had blurry vision for a week or two but was
not hospitalized and had no eye surgery. He denies any
steroid use. IOP is 34 mmHg OD with normal corneal
pachymetry. There is an inferior arcuate scotoma on HVF.
Question 7. What is the diagnosis at this stage?
Gonioscopy of the angle shows:
• Plateau-iris syndrome.

Question 8. What is the pathophysiology of this condition?


• The pathophysiology of Plateau-Iris Syndrome may be
described as an anteriorly rotated ciliary process. Through
this, the peripheral iris is pushed forward.
• This results in a dislocation of the deep chamber centrally,
with the flat iris contour having a sharp drop-off
peripherally.
• The trabecular meshwork would ultimately be occluded
once dilation occurs due to the peripheral iris folding into
the angle. No pupillary block. Question 3. What finding is present?
Question 9. How would you manage this condition? • Angle recession, which is a tear in the ciliary body
between the longitudinal and circular fibers of the muscle.
• Laser iridoplasty and miotics.
Question 4. What is the final diagnosis?
• Angle recession glaucoma.

Question 5. What treatment would you recommend?


• Initial treatment with topical medications. Laser
trabeculoplasty has a poor effect on angle recession, so
surgery is usually considered as the next step.

S/P Laser iridotomy Question 6. What surgical options are best for this type of
disease?
CASE 12: ANGLE RECESSION GLAUCOMA • Trabeculectomy with antimetabolite or glaucoma drainage
A 38-year-old man comes in for a complete eye implant.
exam. He says his vision has always been good and his last
exam was more than 10 years ago. Exam shows 20/20
vision in both eyes, a peripheral posterior

TRANS BY: VIRATA AND SANTOS Page 47 of 51


OPHTHA FINALS TRANS

CASE 13: RUBEOSIS IRIDIS • May require treatment of increased IOP (do not use miotic
A 63-year-old man presents with decreased vision agents or prostaglandin analogues, and avoid carbonic
in the right eye. Examination on the slit lamp showed the anhydrase inhibitors in patients with sickle cell disease),
following (see photo): consider aminocaproic acid
• Daily observation for the first 5 days to monitor IOP and
check for rebleed
• Avoid aspiring containing products

Non-Pharmacological Treatment
• Patient should remain at bedrest
• Sleep with the head slightly elevated
• Provide a shield or patch for the eye
• Anterior chamber washout may be recommended

Question 1. What is the diagnosis? Question 8. What is an 8 ball hyphema?


• Rubeosis iridis (Iris Neovascularization) • A hyphema that has clotted and appears black or purple
owing to impaired aqueous circulation and deoxygenated
Question 2. What is the etiology? blood, which prevents resorption.
• Ocular ischemia, most commonly due to proliferative
diabetic retinopathy, central retinal vein occlusion, and Question 9. What are the indications for anterior chamber
carotid occlusive disease. washout?
• Rubeosis is also associated with anterior segment • Anterior chamber washout is performed for corneal
ischemia, chronic retinal detachment, tumors, sickle cell bloodstaining, uncontrolled elevated IOP, persistent
retinopathy, and chronic inflammation. blood clot, and rebleed.
CASE 14: CENTRAL SEROUS RETINOPATHY
Question 3. What other findings would you look for on A 44-year-old man is worried about blurry vision in the
exam? right eye for the past 2 weeks.
• RAPD
• Increased IOP
• Corneal edema
• Ange neovascularization
• Retinal neovascularization/hemorrhages
• Optic nerve cupping
• Fluorescein angiogram may demonstrate retinal
nonperfusion and neovascularization
• Visual field testing may show glaucomatous defects

Question 4. What is the treatment? 1. WHAT IS THE DIFFERENTIAL DIAGNOSIS?


• Treatment for rubeosis iridis (iris neovascularization) • Central serous chorioretinopathy (CSC)
includes laser photocoagulation for retinal ischemia • Inflammatory choroidal disorders (VKH syndrome)
and possible peripheral cryotherapy. • Uveal effusion syndrome
• Treatment of increased IOP or glaucoma may be • Optic nerve pit
necessary. • Choroidal tumor
• Vitelliform macular detachment
Question 5. What are the complications of this condition? • Pigment epithelial detachment from other causes including
• Neovascular glaucoma (NVG) and hyphema. choroidal neovascularization (CNV)
• If the underlying cause is PDR (proliferative diabetic
retinopathy), then vitreous hemorrhage and tractional
2. WHAT OCULAR EXAM AND ANCILLARY TESTS WOULD
retinal detachment may occur.
BE HELPFUL?
Optical Coherence Tomography (OCT)
Question 6. What are the mechanism and treatment for
• Reveals subretinal fluid including checking for macular
this condition?
schisis or optic nerve excavations seen with optic nerve pit,
Mechanism thickened choroid seen in with VKH, central serous
● Neovascular glaucoma (NVG) is a form of secondary angle- chorioretinopathy, uveal effusion syndrome, pigment
closure glaucoma. epithelial detachments, and retinal atrophy following
● Neovascularization of the iris and angle results in the chronic disease or the characteristic OCT appearance of
occlusion of the trabecular meshwork. vitelliform lesions.

Treatment Fluorescein Angiography


● NVG usually requires a glaucoma drainage implant or • Useful to rule out choroidal neovascularization (rare in this
cyclodestructive procedure to adequately control IOP. age group) & to visualize the hyperfluorescence early with
late pooling of a pigment epithelial detachment & subretinal
Question 7. What is the treatment of hyphema? fluid.
• Topical steroids and cycloplegic

TRANS BY: VIRATA AND SANTOS Page 48 of 51


OPHTHA FINALS TRANS

Indocyanine Green (ICG) angiography Question 2. What other findings may be present?
• Used if both OCT & FA fail to determine the diagnosis Other findings that may be seen:
• Shows hyperfluorescence with late staining in CSC, • Ciliary injection
vascularity with tumors, and rule out CNV. • Anterior chamber cells and flare
• Hypopyon
OCT angiogram (OCTA) • Iris nodules
● Can be useful to also rule out CNV. • Rubiosis
• Synechiae
Additional information: the FA shows: • Increase or decrease in IOP
• Cataract
• Pars planitis
• Optic nerve hyperemia
• Chorioretinitis
• Periphlebitis
• Cystoid Macular Edema

Question 3. How would you work up a patient with


granulomatous uveitis?
Laboratory tests may be needed in order to identify the cause of
granulomatous uveitis, most especially in patients that does not
QUESTION 3. WHAT IS DEMONSTRATED AND WHAT IS reveal significant information from history and physical exam.
THE DIAGNOSIS?
Laboratory work up includes the following:
• The test being shown is fluorescein angiography
• Complete blood count with differential count
displaying an ink blot pattern or a classic “smokestack”
leakage into the pigment epithelial detachment • Erythrocyte sedimentation rate
characteristic of CSC; however, this classic appearance is • Lysozyme
seen in only about 10% of cases. • For detection of syphillis
o VDRL or rapid plasma reagin
o Treponemal antibody absorption test (FTA-ABS) or
QUESTION 4. HOW WOULD YOU MANAGE THIS PATIENT? microhemagglutination assay for T. pallidum
• Observe initially since most of the cases resolve • For detection of toxoplasmosis
spontaneously over 6 weeks. o ELISA or Immunofluorescence assay for IgM and IgG
• Off-label, anti-mineralocorticoid oral therapy titers
(spironolactone or eplenerone) • For sarcoidosis
• Off-label rifampin o Angiotensin-converting enzyme
• Laser treatment or verteporfin (visudyne) • Other lab tests may be necessary depending on the history
• Ocular photodynamic therapy (PDT) of the patient:
o Can be considered for patients who require more
o Antinuclear antibody (ANA)
rapid visual recovery because of occupational
o Rheumatoid factor
reasons, poor vision in the fellow eye due to CSC, o HIV antibody test
no resolution of fluid after several months, o Chest X-ray or CT Scan for sarcoidosis/TB
recurrent episodes with poor vision, or in severe
o Urinalysis
forms of CSC.
o Sacroiliac radiographs for ankylosing spondylitis
• Treatment reduces the duration of symptoms but does not
affect the final visual acuity. Describe the picture
CASE 15: TOXOPLASMOSIS CHORIORETINITIS
A 42-year-old woman reports pain, photophobia,
redness, and decreased vision for 4 days. She recalls
having had a similar episode several years ago.

• There is presence of necrosis which can be described as


whitish fluffy lesion surrounded by retinal edema.
• A peripapillary scar is also seen which signifies that the
lesion is in the healing process.
Question 1. What finding is shown, and what is the • Inflammatory cells are also seen in the vitreous.
diagnosis? • The optic disk is also a bit obscured.
• The most blatant characteristics that can be seen in this • There is presence of white retinitis (active infection) and
image is the presence of large, mutton-fat keratic black scar (previous infection) may indicate
precipitates. This shows a sign of granulomatous uveitis. toxoplasmosis chorioretinitis.

TRANS BY: VIRATA AND SANTOS Page 49 of 51


OPHTHA FINALS TRANS

Question 5. What is the diagnosis? macular edema (diffuse vs focal). OCT angiography
Toxoplasmosis chorioretinitis (OCTA) can show retinal ischemia and microaneurysms.
• Toxoplasmosis infection is the most common cause of Question 4. What level (stage) of disease does this patient
posterior uveitis and focal retinitis. exhibit?
• Diabetic retinopathy can be classified based on the clinical
Question 6. What is the treatment? features. This patient has very severe nonproliferative
• Treatment for this kind of disease includes topical diabetic retinopathy defined by the "4-2-1 rule":
steroids and cyclopegic are prescribed to treat the • (4) intraretinal hemorrhages and/or microaneurysms in all
anterior inflammation. 4 quadrants; or
• Systemic steroids are further added to address the • (2) venous beading in at least 2 quadrants; or
posterior pole lesions or those with intense inflammation. • (1) intraretinal microvascular abnormality (IRMA) in at
• Small peripheral lesions are often observed since they least 1 quadrant.
often heal spontaneously especially in immunocompetent • Very severe NPDR exists if there is more than one these
individuals. features, as in this case.
• However, if patient has decreased vision, moderate to
severe vitreous inflammation, or lesions that threaten the Question 5. How would you manage this patient?
macula, papillomacular bundle, or optic nerve he/she Medical
should be treated 4-6 weeks with antibiotics that kill • The Diabetes Control and Complications Trial (DCCT) and
tachyzoites in the retina. (note: they do not affect cysts) United Kingdom Prospective Diabetes Study (UKPDS)
• Most patient respond well to trimethoprim- concluded that tight blood sugar and blood pressure control
sulfametoxazole (Bactrim). slowed progression of retinopathy, development of macular
• For aggressive lesions or posterior pole lesions, triple edema, need for treatment, and other microvascular
therapy can be considered with pyrimethamine (Daraprim), complications.
folinic acid (leucovorin), and one of the following:
sulfadiazine, clindamycin, clarithromycin, azithromycin, or Anti-VEGF
atovaquone. • For macular edema involving the fovea (center-involving),
• Immunocompromised patients and high-risk patients may ranibizumab, aflibercept and bevacizumab have been shown
require prophylactic treatment. to be very effective therapies.
• This patient would be an ideal candidate for intravitreal anti-
CASE 16 VEGF therapy Protocol T suggests that if the vision was
A 36-year-old man presents with decreased vision worse than 20/50 then aflibercept may be the best choice,
in his left eye. but if the vision was better, then any of the three anti-VEGF
agents would work.
• In year 2 of Protocol T, there was no difference between
aflibercept and ranibizumab, but bevacizumab was not as
effective.
Laser
• The Early Treatment Diabetic Retinopathy Study (EDTRS)
concluded that focal/grid laser photocoagulation decreased
moderate visual loss by 50% in patients with clinically
significant macular edema (CSME) defined as:
• (1) retinal thickening within 500 um of the macular center
or
• (2) hard exudates within 500 um of the macular center
Question 1. What are the retinal findings? with adjacent thickening or
• Numerous intraretinal hemorrhages, microaneurysms, • (3) zone of retinal thickening 1 disc area in size any
cotton-wool spots, and lipid exudates scattered throughout portion of which is within 1 disc diameter of the macular
the posterior pole. center.
• No retinal or optic nerve neovascularization, preretinal • CSME is based only on clinical examination and not visual
hemorrhage, or vitreous hemorrhage is seen. acuity (treat even with 20/20 vision) or other imaging studies.
Laser treatment is second line to anti-VEGF therapy.
Question 2. What is the most likely diagnosis and what
Steroids
tests can be performed to confirm it?
• Considered third-line therapy behind laser and anti-VEGF
• Diabetic retinopathy.
therapy, intraocular steroids have been shown to be effective
o Check serum hemoglobin A1c
especially in patients who are already pseudophakic.
o fasting blood sugar
o blood pressure • Although the Diabetic Retinopathy Clinical Trials Network
(DRCR.net) Protocol B did not find steroids better than laser
Question 3. What additional ophthalmic tests would be in the overall study population, DRCR.net Protocol I did show
helpful? a benefit in pseudophakic patients.
• Similarly, the sustained release, fluocinolone acetonide,
• A fluorescein angiogram (FA) would be useful to evaluate
steroid implant (Iluvien) has been shown to be effective over
macular ischemia and to rule out neovascularization.
a 3-year follow-up period in patients who had previous laser
• An optical coherence tomography (OCT) scan would be
therapy and persistent edema (FAME Study).
useful to evaluate for the presence of posterior hyaloidal
traction, epiretinal proliferation, and the nature of the

TRANS BY: VIRATA AND SANTOS Page 50 of 51


OPHTHA FINALS TRANS

• Finally, the dexamthesone implant (Ozuredex) has been


found to be effective (MEAD Study). With all steroids, careful
monitoring of cataract formation and IOP is important.
Surgery
• If a patient exhibits posterior hyaloidal traction then pars
plana vitrectomy with membrane peel should be considered.
This patient did not exhibit any traction.

Question 6. This patient has an even more advanced


disease in his fellow eye. How would you treat it?
• The first question would be to decide if the patient had high-
risk characteristics or not.
• in the presence of high-risk characteristics, immediate
panretinal photocoagulation (PRP) should be instituted.
• High-risk characteristics of PDR are defined as:
neovascularization of the disc (NVD) > standard photo 10A
used in DRS (one-third to one-quarter disc area) or any NVD
and VH or preretinal hemorrhage or neovascularization
elsewhere (NVE) > standard photo 7 (one-half disc area) and
VH or preretinal hemorrhage.

END OF TRANS!

Special thanks to my ophthalmology transers!

TRANS BY: VIRATA AND SANTOS Page 51 of 51

S-ar putea să vă placă și